You are on page 1of 107

Vicealmirante Franklin M.

Zeltzer Malpica
RECTOR

SISTEMA DE APRENDIZAJE AUTOGESTIONADO ASISTIDO (TRIPLE A)

Dra. Rosa M. Puerta Castro


Coordinadora General del Sistema Triple A

PRODUCCIÓN DE MATERIALES DIDÁCTICOS DE LA ASIGNATURA


“FUNDAMENTOS DE MATEMÁTICA”

Dra. Rosa M. Puerta Castro


Coordinadora General del Sistema Triple A

Lic. José Gómez Barrios (Núcleo Caracas)


Lic. José Santamaría (Núcleo Caracas)
Prof. Alberto Ochoa Parada (Triple A Sede)
Especialistas de Contenido

Prof. Alberto Ochoa Parada (Triple A Sede)


Diseñador Instruccional

Lic. Guillermina Indriago (Triple A Sede)


Especialista de Redacción y Estilo

Prof. Alberto Ochoa Parada (Triple A Sede)


Diseñador y Diagramador

PRODUCCIÓN GRÁFICA E IMPRESIÓN


(NOMBRE DE LAS EMPRESAS)

Todos los derechos reservados.


Prohibida la reproducción total o parcial por cualquier medio gráfico, audiovisual o computarizado
de este material didáctico sin previa autorización escrita

2007 Universidad Nacional Experimental Politécnica de la Fuerza Armada (UNEFA)

Av. La Estancia con Av. Caracas y Calle Holanda frente al Edificio Banaven (Cubo Negro), Chuao.
Código Postal 1061
Caracas, Venezuela

http://www.aaa.unefa.edu.ve

2
ÍNDICE DE CONTENIDO
INTRODUCCIÓN 5

UNIDAD Nº 1: EXPRESIONES ALGEBRAICAS 7

LECTURA Nº 1:Cambio de Paradigma 7

LECTURA Nº 2:¿Qué es el Número? 8

LECTURA Nº 3:Los Números Cuadrados 9

LECTURA Nº 4:Los Polinomios 10

LECTURA Nº 5:Productos Notables 22

LECTURA Nº 6:La Factorización como Herramienta de Simplificación 32

LECTURA Nº 7:¿Cómo Completar Cuadrados? 33

LECTURA Nº 8:Métodos de Factorización 34

UNIDAD Nº 2: VALOR ABSOLUTO E INECUACIONES 42

LECTURA Nº 9:Numeración Antigua Egipcia 42

LECTURA Nº 10:El Valor Absoluto y los Números Reales 43

LECTURA Nº 11:Los Intervalos y el Calendario 45

LECTURA Nº 12:Inecuación contra Ecuación 46

LECTURA Nº 13:Conociendo las Inecuaciones 47

LECTURA Nº 14:Inecuaciones en la Recta 52

UNIDAD Nº 3: GEOMETRÍA Y TRIGONOMETRÍA 56

LECTURA Nº 15:Algunos Sistemas de Medida 56

LECTURA Nº 16:El Sistema Métrico Decimal 57

LECTURA Nº 17:Figuras Poligonales 62

LECTURA Nº 18:Los Triángulos, los Cuadriláteros y sus Relaciones 63


Métricas

LECTURA Nº 19:La Circunferencia y sus Elementos 67

3
LECTURA Nº 20:Cuerpos Geométricos y sus Elementos 69

LECTURA Nº 21:El Número Pi (∏) y el Cálculo de Áreas 72

LECTURA Nº 22:Thales y la Pirámide de Keops 79

LECTURA Nº 23:La Trigonometría 82

LECTURA Nº 24:La Trigonometría ¿para qué sirve? 86

LECTURA Nº 25:Teorema de Pitágoras 87

UNIDAD Nº 4: PLANO CARTESIANO, RELACIONES Y FUNCIONES 89

LECTURA Nº 26:El Plano Cartesiano 89

LECTURA Nº 27:Coordenadas y Tecnología 91

LECTURA Nº 28:Funciones que tienen Historia 92

LECTURA Nº 29:La Función Lineal 92

LECTURA Nº 30:Distancia entre dos Puntos en el Plano 94

LECTURA Nº 31:Clasificación de Funciones 96

4
INTRODUCCIÓN
La Universidad Nacional Experimental Politécnica de la Fuerza Armada (UNEFA), te da la
bienvenida al primer semestre de tu carrera y a la vez te ofrece este valioso recurso que
contiene una selección de lecturas para la asignatura Fundamentos de Matemática.
La Matemática en todas las sociedades civilizadas, ha sido y es,
un instrumento imprescindible para el conocimiento y
transformación de la realidad que caracteriza la acción humana;
por ello, es considerada como ciencia prototípica del
razonamiento, constituye un conjunto amplio de modelos y
procedimientos de análisis, cálculo, medida y estimación de las
relaciones necesarias entre los diferentes aspectos de la realidad.
A semejanza de otras disciplinas conforma un campo en continua
expansión y creciente complejidad, donde los constantes avances
dejan atrás las acotaciones y concepciones tradicionales. A
medida que avanza la tecnología, la sociedad del conocimiento y la información, la
matemática estudia cada vez objetos más abstractos, sus relaciones cuantitativas y
formas espaciales.
En el desarrollo del aprendizaje matemático, las operaciones mentales concretas como:
contar, ordenar, comparar, clasificar, relacionar, analizar, sintetizar, generalizar, abstraer,
entre otras, aunado a la experiencia y la inducción, desempeñan un papel de primer
orden, pues te permiten construir representaciones lógicas y matemáticas que más tarde
tendrán valor por sí mismas de manera abstracta y serán susceptibles de materializar en
un sistema plenamente deductivo, partiendo de la experiencia directa. De ahí, que la
eficacia de la matemática radica en la precisión de sus formulaciones y sobre todo en la
aplicación consecuente del método hipotético-deductivo, característico de esta ciencia.
De las reflexiones anteriores, se puede inferir que durante el estudio de la Matemática se
presentan exigencias para el uso y desarrollo del intelecto, mediante la práctica deductiva
y la representación mental de relaciones espaciales; ella hace una contribución esencial
al desarrollo de tu pensamiento. Se puede plantear que el pensamiento matemático hoy
en día, es un componente vital e influyente en cada uno de los aspectos de la cultura
universal.
El desarrollo intelectual que se logra por de la enseñanza de la Matemática, se promueve
debido a que:
∗ Los conceptos, las proposiciones y los procedimientos matemáticos, poseen un
elevado grado de abstracción y su asimilación obliga a los alumnos a realizar una
actividad mental rigurosa.
∗ Los conocimientos matemáticos, están estrechamente vinculados, formando un
sistema que encuentra aplicación práctica de diversas formas, lo cual permite buscar
y encontrar vías de solución distintas, por su brevedad, por los medios utilizados o la

5
ingeniosidad de su representación. Ello te ofrece un campo propicio para el desarrollo
de la creatividad y el pensamiento lógico.
Las formas de trabajo y de pensamiento matemático requieren una constante actividad
intelectual, que te exige analizar, comparar, fundamentar, demostrar y generalizar, entre
otras operaciones mentales.
En esta selección de lecturas, encontrarás un resumen de los contenidos básicos de los
temas que componen la asignatura, los cuales te apoyarán en el logro de un aprendizaje
de calidad.
Los temas seleccionados, complementan los ya estudiados durante el desarrollo de
Curso de Inducción Universitaria, con miras a integrar todos los contenidos básicos de la
asignatura necesarios para emprender una carrera universitaria.
Durante la primera unidad, se trabajará con las expresiones algebraicas, específicamente
las operaciones con polinomios, los productos notables más comunes y los métodos
de factorización, considerando que aparte de los cálculos elementales, como la adición,
la multiplicación y la potenciación entre otras, aplicables en todas las ramas de la
matemática, y a través de propiedades de composición bien definidas, se derivan
procedimientos que permiten simplificar con mayor facilidad las operaciones indicadas
tales como: el producto notable y la factorización, que son herramientas muy prácticas
para la agilización en la búsqueda de un resultado concreto a los problemas
matemáticos.
La segunda unidad está referida a los temas de valor absoluto e inecuaciones en
donde se plantean ejercicios y problemas relacionados con la realidad y cuya solución
involucra las propiedades que rigen las desigualdades entre dos o más expresiones
algebraicas.
La tercera unidad contempla los temas de Geometría y Trigonometría como vínculo el
más real entre la abstracción que representa la matemática con el mundo que nos rodea;
se establece una relación entre las figuras planas y los cuerpos geométricos con los
cálculos que de ellos se derivan. Involucrando las gran diversidad de unidades de
medida con sus respectivas conversiones.
Se culmina con la cuarta unidad; ella contiene los temas relacionados al plano
cartesiano, relaciones y funciones como manera de utilizar los conocimientos previos
en la solución de problemas algebraicos y su respectiva representación gráfica.

6
UNIDAD 1
EXPRESIONES ALGEBRAICAS
LECTURA Nº 1: CAMBIO DE PARADIGMA

Tomado con fines instruccionales de:


Ascanio, R. y González, P. (2004). “Cambio de
Paradigma”, Homotecia: Paradigmas. Publicación
periódica Nº 6. Año 2. Valencia:.

Un grupo de científicos colocó cinco monos en una jaula, en cuyo centro había una
escalera y, sobre ella, un racimo de cambures. Cuando un mono subía por la escalera
para agarrar los cambures, los científicos lanzaban un chorro de agua fría sobre los que
quedaban en el suelo. Después de algún tiempo, cuando un mono iba a subir la escalera
los otros lo agarraban a golpes evitando así el castigo con el agua fría.
Ya transcurrido un tiempo más, ningún mono subía la escalera, a pesar de la tentación de
los cambures. Entonces, los científicos decidieron sustituir uno de los monos. La primera
cosa que hizo fue subir por la escalera, siendo rápidamente bajado a golpes por los
demás monos, quienes le pegaron sin contemplación alguna. Después, de algunas
palizas, el nuevo integrante del grupo ya no subió más la escalera.
Luego, un segundo mono fue sustituido, y ocurrió el mismo espectáculo que la vez
anterior. EL primer sustituto participó con entusiasmo en la paliza al recién llegado. Un
tercero, de los más antiguos, fue cambiado y volvió a repetirse el mismo suceso. Y así
pasó cuando cambiaron al cuarto de los primeros cinco monos, y finalmente el último de
los veteranos que también fue reemplazado.
Los científicos quedaron, entonces, con un grupo de cinco monos que, aun cuando nunca
recibieron un baño con agua fría, continuaban, sin ninguna explicación, golpeando a
aquel que intentase llegar a los cambures.
Si fuese posible preguntar a alguno de los miembros del grupo por qué pegaban a quien
intentase subir por la escalera, con certeza la respuesta sería: “No sé, las cosas aquí
siempre se han hecho de esa manera…”. ¿Te parece familiar la respuesta?
Vamos a reflexionar un poco, nos hemos preguntado alguna vez el por qué estamos
golpeando… y… por qué estamos haciendo las cosas de una manera, si a lo mejor las
podemos hacer de otra.
En muchas ocasiones, por no decir siempre, vemos como se asume un rechazo total al
estudio de las matemáticas por parte de los estudiantes, quizá parte de esa actitud es
producto de la concepción negativa y experiencias ajenas que han pasado de unos a
otros, sin detenerse a pensar y a averiguar que tan cierto es y que tan malo puede ser,
apartando radicalmente la posibilidad de poseer una herramienta que nos puede guiar al

7
éxito en cualquier ámbito de nuestra cotidianidad, ya sea en las clases, el hogar, el
trabajo, de compras, entre otros.

LECTURA Nº 2: ¿QUÉ ES EL NÚMERO?


Tomado con fines instruccionales de:
Gómez, J. (2006). ¿Qué es el número? Artículo no
publicado (pp.1-2). Tinaquillo, Estado Cojedes.

A lo largo de nuestra vida escolar, nos hemos enfrentado a toda clase de operaciones con
los números; los multiplicamos, los sumamos, los potenciamos, nos ayudan a ordenar,
clasificar y muchas otras operaciones que faltan mencionar. Para muchos de nosotros, es
sencillo y elemental contar con ellos; pero cabe preguntar: ¿podemos definirlos?..., allí
es donde está la dificultad, cuando tenemos que dar una definición formal de Número.
¿Te lo has preguntado alguna vez?
Desde muy niños, nos enseñan a contar con los dedos; luego, con objetos;
posteriormente, los aprendemos a escribir y ordenar. En nuestra prosecución académica,
los utilizamos en las primeras operaciones fundamentales, unidades, decimales,
propiedades, en fin, un mundo complejo en función a ellos; pero insisto… ¿Quién en
algún momento te enseñó a definirlos? La creación del número es una de las más
grandes hazañas de la mente humana y desde tiempos inmemoriales a la matemática se
le atribuye el reinado de las ciencias puras y exactas; sin embargo, qué tan difícil sería
aprender a definir “Número”.
Algunos autores aseguran que “número es todo aquello que es el número de una clase”
y la definición de Russell (1988 ) se centra en que “número es todo aquello que es el
número de un conjunto”… sea cual sea la definición pareciera redundar pero, si nos
detenemos a reflexionar, ellas nos aproximan a una realidad que aparenta estar sólo en
nuestra mente, aunque cada uno de nosotros podemos vivir a diario y relacionarlo con el
entorno. Veamos otro ejemplo: Si poseemos un conjunto o clase de elementos llamados
balones, podemos afirmar que la clase es el nombre del objeto y/o sujeto y que el número
es la cantidad de balones que existan en esa clase.
Ejemplo:

Número: 8
Figura 1
Clase: Balones

En conclusión, “ balones” refiere al número de elementos que se encuentran en el


conjunto o clase de balones. Este ejemplo y futuras comparaciones nos pudiesen resultar
8
muy obvias, pero ¿sabes algo? en la matemática, nada es obvio… a partir de este
momento conociste una de las definiciones de la matemática que mucho se utiliza, pero
poco se reflexiona sobre ella. Esta abstracción, debe surgir de las necesidades primarias
que tenemos cada uno de nosotros de ordenar, clasificar, seriar y establecer relaciones
con el medio.
Referencia:
Russell, B. (1988). Introducción a la filosofía matemática. Paidós Estudio Básica. (p.25)
Barcelona, España

LECTURA Nº 3: LOS NÚMEROS CUADRADOS


Tomado con fines instruccionales de:
Suárez, E. y Cepeda, D. (2003).
Matemática para Educación Básica.
Editorial Santillana, S.A. (p. 68). Caracas,
Venezuela.

Los números cuadrados o cuadrados perfectos (1, 4, 9, 16…) fueron llamados así por
primera vez por los pitagóricos, una orden comunal fundada por Pitágoras (siglo VI a.C.
en la costa suroeste de Italia), donde la matemática regía los principios de convivencia
entre los miembros.
Estos números indican cantidades o clases de objetos que pueden agruparse formando
un cuadrado (ver figura). Además de los números cuadrados, los pitagóricos definieron
otros “números figurados”, como los triangulares o pentagonales.
Observa el esquema de los números cuadrados perfectos:

Figura 2

Fíjate que cada cuadrado perfecto es igual a la suma de cierta cantidad de números
impares consecutivos:
12 = 1 (un número impar)
22 = 1 + 3 (dos números impares)
32 = 1 + 3 + 5 (tres números impares)
42 = 1 + 3 + 5 + 7 (cuatro números impares)

9
Así como en la ciencia matemática existen entes de características perfectas, también
nosotros, mediante el esfuerzo, la dedicación, la práctica constante y la pasión por lo que
hacemos, debemos ir perfeccionando nuestras competencias y habilidades en pro de una
mayor satisfacción tanto intelectual como personal.

LECTURA Nº 4: LOS POLINOMIOS


Tomado con fines instruccionales de:
Santamaría, J. (2006). Los polinomios. Artículo no
publicado (pp.1-20). Tinaquillo, Estado Cojedes.

En estudios anteriores has trabajado con operaciones de suma, resta, multiplicación y


división de números naturales, enteros, racionales e irracionales. Este estudio se
enmarca dentro de la aritmética, rama de la matemática que se encarga de situaciones
específicas, donde las operaciones sólo se hacen con números.
Si profundizamos un poco más en nuestra experiencia, ya sea la que obtuvimos en el
bachillerato o en cualquier otra actividad escolar, es posible que recordemos algún
conocimiento sobre las operaciones con polinomios, donde de manera similar aplicabas
la suma, resta, multiplicación y división, pero ya no sólo intervenían números sino que
también se involucraban letras. El estudio de la matemática se tornaba un poco más
abstracto, pues aquellas situaciones específicas que se trabajaban en aritmética ahora
tomaban un carácter de generalización, es decir, podían representar situaciones diversas
en un mismo campo. Ahora la matemática se enfoca desde Álgebra.
A pesar de tener más o menos claro las distintas operaciones con polinomios, es
necesario retomar y practicar esos conocimientos hasta dominarlos por completo, pues
de ello depende alcanzar las competencias en contenidos pertinentes a la asignatura,
como lo son: las inecuaciones y las funciones; además de otras actividades que guardan
relación con este tema.
Empecemos definiendo lo que es un polinomio; este término es de origen griego “poli”
que significa muchos y “nomio” expresión algebraica. Un polinomio, matemáticamente
hablando es una suma algebraica de varias expresiones algebraicas, que representan
cantidades desconocidas. Cuando decimos suma algebraica nos referimos a una
operación combinada, donde intervienen la suma y la resta, y al hablar de expresiones
algebraicas significa los términos que componen la suma. Cada término que compone un
polinomio es una estructura matemática que consta de una parte numérica y una parte
literal.
Exponente de la variable
Ejemplo de la Estructura de una Término:

− 3x 5
Parte numérica o coeficiente de la variable Parte literal o variable

10
CARACTERÍSTICAS DE UN POLINOMIO:
3 2 1
Sea el polinomio: x − 5x 3 + x −
4 3
Vamos a ordenarlo por el exponente de la variable y a describir sus elementos:
3 2 1
− 5x 3 + x +x−
4 3
Términos − 5x 3 3 2 x 1
x −
4 3
Variable x x x
Coeficientes de la variable −5 3 1
4
Exponentes de la variable 3 2 1
* Grado del polinomio 3
Término Independiente 1

3
*El grado del polinomio lo representa el exponente mayor de la variable
Clasificación de los Polinomios
Los polinomios, según el número de términos, se clasifican en:
- Monomio: Es aquella expresión algebraica que consta de un solo término.
3 2 a 2 bx
Ejemplos: − x +5
7 2
- Binomio: Es aquella expresión algebraica que tiene dos términos:
5
Ejemplos: 3x + 1 x4 − a a+b
4
- Trinomio: Es aquella expresión algebraica que tiene tres términos:
6 3 1 −9 2
Ejemplos: x +x− y + y −5
5 7 2
- Polinomio: Es aquella expresión algebraica que tiene más de tres términos:
3 4 2 3
Ejemplo: − x + x − x2 +1
4 5
Recuerden que los términos en un polinomio se identifican porque están separados unos
de otros por el signo positivo (+) o el negativo (-).

OPERACIONES CON POLINOMIOS

Anteriormente se dijo que con las expresiones algebraicas, se cumplen las operaciones
de adición, sustracción, multiplicación y división. Vamos a trabajar cada operación y
aprender un poco más de ellas.
11
Adición de polinomios: La adición consiste en reunir dos o más expresiones
algebraicas, llamadas sumandos, en una sola que se le llama suma.
En la aritmética la adición siempre significa aumento, pero en el álgebra es un concepto
más general por lo que puede significar aumento o disminución.
En una adición de polinomios se puede dar una agrupación de términos semejantes.
Incluso, hasta un polinomio puede tener inmerso términos semejantes.
Hay semejanza entre términos cuando:
 Tienen la misma variable o variables.
 Tienen igual exponente en la variable o variables.
Ejemplo: Aunque los coeficientes
Son términos semejantes: de las variables sean
diferentes
− 5x 2 + 3x 2 + x 2

La variable “x” es la misma El exponente “2” de la


para los tres términos variable es igual para los
tres términos

Entonces, se puede hacer una agrupación con estos términos y reducirlos a una sola
expresión aplicando una suma.
Ejemplo Nº 1:
Eliminando los paréntesis queda:
−5 x 2 +3 x 2 + x 2 =
Tomemos los coeficientes formando una suma indicada con ellos y esto lo multiplicamos
por la variable con su respectivo exponente, así:
( − 5 + 3 + 1) ⋅ x 2 =
Efectuamos la suma algebraica entre las cantidades que están dentro del paréntesis:

( − 5 + 3 + 1) ⋅ x 2 = ( − 5 + 4) ⋅ x 2 Primero sumamos los enteros positivos 3 y 1

( − 5 + 3 + 1) ⋅ x 2 = ( − 1) ⋅ x 2 Se restan las cantidades por ser de signos diferentes y la


diferencia lleva el signo de la mayor (-5 y -4)
( − 5 + 3 + 1) ⋅ x 2 = − 1 ⋅ x 2 Se elimina el paréntesis

( − 5 + 3 + 1) ⋅ x 2 = − x 2 Como el 1 es elemento neutro de la multiplicación, sólo se


multiplican los signos (+ . - = -)

2
Son términos no semejantes los siguientes: 6x 3 , 6x 2 , 6 y ,

12
Los términos 6x 3 y 6x 2 , tienen igual variable pero distintos exponentes, y a pesar que
2
tienen el mismo coeficiente no son términos semejantes. El término 6 y no es semejante
a ninguno de los otros dos términos, pues su variable es distinta.

Veamos algunos ejemplos de adición de polinomios:


Cuando es una suma de monomios
Ejemplo Nº 2:
Sumar: − 5x 2 y 7 x Observa que, como los términos
no son semejantes la suma se deja
Solución: − 5 x 2 + 7 x = −5 x 2 + 7 x indicada

Cuando es una suma de binomios


3 2 1 7 2
Ejemplo; Sumar: x − y x + 3x
4 3 8
 3 2 1 7 2  Indicamos la operación de los dos
Solución:  x −  +  x + 3x  = binomios agrupando cada uno entre
4 3  8  paréntesis

3 2 1 7 2 Eliminamos los paréntesis, como el signo


x − + x + 3x = que los precede es positivo, no se afecta
4 3 8
ningún término

3 2 7 2 1
 x + x  − + 3x = Agrupamos los términos semejantes
4 8  3

Extraemos la variable con su respectivo


3 7  1
 +  x 2 + 3x − = exponente como factor dejando los
4 8  3 coeficientes dentro del paréntesis.

Observe que estos nos indican una suma
de fracciones con diferente denominador

Recordar 1- m.c.m(4,8) = 8
:
Para sumar fracciones de diferente 3 7
denominador 2- + =
4 8 8
Se calcula el mcm entre los denominadores
3- 3 24
Esta cantidad es el denominador del resultado 8. = =6
Se multiplica cada fracción por el mcm y estas 4 4
cantidades forman el numerador del resultado 7 56
Se efectúa la operación indicada y obtenemos la
8. = =7
8 8
fracción resultado
4- 3 7 6 + 7 13
+ = =
4 8 8 8

13
Luego el polinomio resultante es:
13 2 1
x + 3x −
8 3

En la adición de trinomios y polinomios se procede igual que en las sumas anteriores,


solo debes estar pendiente de la agrupación de términos semejantes. Es importante
señalar que la sustracción de polinomios es un caso particular de la adición. Esto lo
podemos explicar de la siguiente manera:
Ejemplo Nº 3:
3 7 1 1 5
Sea A = x + 6x − y B = x3 − x 2 + x +
5 4 5 2 6
y nos piden determinar: A – B =
3 2 7 1 2 1 5
Es decir, al polinomio x + 6x − le restamos el polinomio x − x + x +
3

5 4 5 2 6
estructuremos la operación:
3 2 7  1 1 5
A− B = x + 6x − −  x 3 − x 2 + x + 
5 4  5 2 6
Observa que el polinomio B por estar precedido del signo negativo se encierra entre
paréntesis.
Recordar: 1 2 1 5
Si B = x − x + x+
Para eliminar signos de agrupación 3

5 2 6
Entonces
Cuando un paréntesis está precedido del signo
menos, todos los términos que están dentro de él 1 2 1 5
cambian de signo − B = −x3 + x − x−
5 2 6
- B es el opuesto de B

Luego, la operación quedaría así:

3 2 7  1 1 5
A + (− B) = x + 6x − +  − x 3 + x 2 − x − 
5 4  5 2 6
Si eliminamos el paréntesis:
3 2 7 1 1 5
A + (− B) = x + 6x − − x3 + x 2 − x −
5 4 5 2 6
Agrupamos los términos semejantes:

3 1   1   7 5
A + (− B) =  x 2 + x 2  +  6 x − x  − x 3 +  − − 
5 5   2   4 6

14
Extraemos la variable de cada paréntesis con su respectivo exponente, dejándola como
factor

3 1  1  7 5
A + (− B) =  +  x 2 +  6 −  x − x 3 +  − − 
5 5  2  4 6
Observa que dentro de cada paréntesis hay una suma de fracciones con diferente
denominador.
Vamos a realizar cada adición por separado:
Recordar:
Para sumar fracciones con igual denominador
3 1 4
1º Adición  +es una
Observa : que  = suma de fracciones con igual
 5 5  5 resultante tendrá el mismo
denominador. La fracción
denominador común y el numerador será la suma de
los numeradores parciales

Recordar:
Para sumar fracciones con diferente denominador 2º Adición
 6 1  12 1 12 − 1 11
Tenemos una suma de fracciones con diferente  + = − = =
denominador, calculamos el m.c.m de los 1 2 2 2 2 2
denominadores; es decir, m.c.m (1,2) = 2, este
m.c.m= 2 representa el denominador común a todas
las fracciones; ahora, los numeradores también
cambian multiplicando el m.c.m= 2 por las
360
fracciones parciales

Recordar:
3º Adición Para sumar fracciones con diferente denominador
Calculamos el mcm entre los denominadores mcm (4
7 5 (12 . 7 / 4) (12.5 / 6)
− − =− − , 6) = 12, este es el denominador del resultado y esa
4 6 12 12 misma cantidad se multiplica por cada fracción para
calcular los nuevos numeradores
7 5 21 10 31
− − =− − =−
4 6 12 12 12

Finalmente, realizadas las adiciones de los términos semejantes, tenemos:


4 2 11 31
A + (− B) = − x 3 + x + x−
5 2 12

Practica la Adición de polinomios con los siguientes ejercicios:


- Sean los polinomios

15
1 2 1 6 3 7
A= x + 6x − , B= x − x 2 + 92 − ,
2 3 7 2

3 1 3 2x 2 8x
C= − x + x2 , D=− + − − x3
5 4 8 9 3
Calcula:
1) A + B + C = 3) (D + A) – C = 5) D + B =
2) D + C + A = 4) B – (D + A) = 6) C – A =

Multiplicación de Polinomios:

La multiplicación de polinomios, es una operación que consiste en multiplicar dos o más


polinomios llamados factores para obtener otro polinomio llamado producto. Para
multiplicar polinomios es necesario tener claro la regla de los signos, las leyes de la
potenciación y la agrupación de términos semejantes.
Recordar: Recordar:
Regla de los signos
Leyes de la potenciación
+*+=+

* *
- -=+ * *
*
* *

Veamos algunos casos de la multiplicación:

Multiplicación de Monomios

Multiplicar: En esta multiplicación tenemos varios factores con


( 3 x ) . ( − 2 x ) . ( − 5) =
2
sus respectivos signos, hay factores numéricos y
factores literales o variables.

Observa que los coeficientes numéricos de cada


monomio, son también factores y se pueden
manipular independientemente de la variable,
( 3) . ( − 2) . ( − 5) . ( x 2 ) . ( x ) = siempre y cuando estén como factores dentro de la
misma multiplicación. En la organización es
conveniente que los factores numéricos sean los
primeros en expresarse.

16
Si multiplicamos los signos de cada uno
( + 3) . ( − 2) . ( − 5) . ( + x 2 ) . ( + x ) = + de los factores: + . - . - . + . + = +
obtenemos el signo del producto. En este
caso es positivo

( 3).( − 2).( − 5).( x 2 ).( x ) = +30.( x 2 ).( x) Ahora calculamos el producto de los factores
numéricos: 3 . 2 . 5 = 30

Para multiplicar las variables (la parte literal),


( 3).( − 2).( − 5).( x 2 ).( x ) = +30.x 3
que son potencias, tienes que estar claro con la
ley de la potenciación que dice que “en la
multiplicación de potencias de igual base se
obtiene otra potencia con la misma base, cuyo
exponente resulta de sumar los exponentes
parciales de cada potencia” x2 . x = x2+1 = x3

Este es el resultado de multiplicar los monomios


(3x ) . ( − 2 x ) . ( − 5) = 30 x
2 3

Multiplicación de Monomios por polinomios

Para multiplicar un monomio por un polinomio, se


3 2  2 5 aplica una propiedad distributiva del producto con
Multiplicar:  x . − x + 2 x −  =
5  2 respecto a la adición, de esta manera obtenemos
una suma algebraica con los productos parciales.

3 2  2 5 3 2 3 2
( )
3  5
 x . − x + 2 x −  =  x . − x +  x .( 2 x ) +   .  − 
2

5  2 5  5  5  2

Observa que cada producto parcial es una multiplicación de dos monomios. Recuerde el
procedimiento para este caso. En cada multiplicación parcial, realiza primero la multiplicación
de los signos, luego, multiplica los coeficientes de cada monomio y por último realiza la
multiplicación de las variables o potencias literales.

17
Vamos a calcular los productos por separado:

1° producto: Coeficientes

3 2
( )
 3 2
( )( )
2 3 4
 x . − x =  .( − 1). x . x = − x
2 Producto
5  5 5

Potencias
Literales

 3  3   1 3 Ya debes tener claro la regla de los signos (+. - = -) ; los


 .( − 1) =  . −  = − coeficientes o parte numérica son números racionales; es decir,
5  5   1 5 fracciones. Para multiplicar fracciones se hace de forma lineal,
numerador por numerador y denominador por denominador.

La multiplicación de las potencias literales se realiza aplicando la


x 2 .x 2 = x 2 + 2 = x 4
ley de potenciación “cuando se multiplican potencias de igual
base, el producto que resulta es otra potencia con la misma base y
el exponente es la suma de los exponentes parciales”.

2° producto:
Se procede igual al caso anterior:
3 2 3 2 2
( ) 6 3
 x .( 2 x ) =  . . x ( x ) = + x  3  3 2
 .( 2 ) =  .  = +
6
Coeficientes
5  5 1 5
5 51 5

x 2 .x = x 2+1 = x 3 Potencias Literales

3° producto: Se procede igual al caso anterior:


 3 5 15 3.5 3
 . −  = − = − =−
 3 2  5   3 5 2
( )
15 2 3 2
 x . −  =  . − . x = − x = − x 5 2 10 2.5 2
5  2 5 2 10 2

Observa que el producto de los coeficientes, resultó una fracción que se simplificó,
debido a que al descomponer tanto el numerador como el denominador, resultó un factor
común (el 5), el cual se canceló por ley de la potenciación, quedando una fracción
irreducible. Luego, reuniendo los productos parciales resultantes conformamos el
producto total de la multiplicación inicial:

3 2  2 5 3 4 6 3 3 2 El polinomio resultante no tiene términos


 x . − x + 2 x −  = − x + x − x semejantes por lo tanto es un polinomio
5  2 5 5 2
irreducible.

18
Multiplicación de un polinomio por otro polinomio
3 4 3 
Multiplicar:  x −  y x 2 − x + 1
2 3 4 
Observa que el primer factor es un polinomio de dos términos, por lo tanto hay que aplicar la
propiedad distributiva dos veces. El primer término del binomio multiplica a todos los
términos del trinomio, luego el segundo término del binomio multiplica a todos los términos
del segundo factor, es decir, del trinomio.

3 4 3 2  Si observas cada par de


Solución:  x − . x − x + 1 líneas notarás como se
2 3 4 
efectuaron los productos

 3  3 2   3  3   4 3 2   4  4
 x . x  +  x .( − x ) +  x .( + 1) +  − . x  +  − .( − x ) +  − .( + 1)
 2  4   2  2   3 4   3  3

1° producto:
Después de aplicar la propiedad
 3  3 2   3 3
( )
9 3
 x . x  =  . .( x ). x = x
2 distributiva hemos obtenido muchos
 2  4   2 4 8 productos parciales, para ser más
exactos, seis productos. Vamos a
2° producto: resolverlos uno a uno:

3  3 3 2
 x .( − x ) =  .( − 1).( x ).( x ) = − x
2  2 2

3° producto:
3  3 3
 x .( + 1) =  .( + 1).( x ) = + x
2  2 2

4° producto:
−4 3 2  −4 3 2
 ⋅ x  = 
12 2
( )
 ⋅   ⋅ x = − x = −x
2

 3  4   3  4 12

5° producto:
−4 −4 4
  ⋅ (− x) =   ⋅ ( − 1) ⋅ ( x ) = + x
 3   3  3

6° producto:
 4  4   1 4
 −  ⋅ ( + 1) =  −  ⋅  +  = −
 3  3   1 3

19
Luego:
Tomamos los productos parciales resultantes y estructuramos el polinomio total.
3 4 3 2  9 3 3 2 3 4 4
 x −  ⋅  x − x + 1 = x − x + x − x + x −
2

2 3 4  8 2 2 3 3

Revisamos si el polinomio resultante tiene términos semejantes; si los tiene hacemos


agrupaciones con ellos:

3 4 3 2  9 3  3 2 2 3 4  4
 x −  ⋅  x − x + 1 = x +  − x − x  +  x + x  −
2 3 4  8  2  2 3  3

Como en los casos anteriores, en agrupaciones de términos semejantes extraemos la


variable con su respectivo exponente como factor fuera del paréntesis.

3 4 3 2  9 3  3  2 3 4 4
 x −  ⋅  x − x + 1 = x +  − − 1 x +  + ⋅x −
2 3 4  8  2  2 3 3
Realizamos la adición dentro de cada paréntesis paso a paso:

 3 1 3 2 −3− 2 5
1º Adición: − −  = − − = =−
 2 1 2 2 2 2

3 4  9 8 17
2° Adición:  + = + =
2 3 6 6 6
Luego, resueltas las adiciones, volvemos al polinomio.
3 4 3 2  9 3 5 2 17 4
 x −  ⋅  x − x + 1 = x − x + x −
2 3 4  8 2 6 3

De esta manera, hemos llegado al producto final de la multiplicación de dos polinomios.


Para que practiques los procedimientos en la multiplicación de polinomios te proponemos
los siguientes ejercicios:
Dadas las expresiones algebraicas:
7 4 8 3 6
P=− x Q= − x2 R= x −x+
2 5 7 7
x2 3 5 11
T = − + 9x − V =
2 4 9 3
Calcula:
1) V .P.Q = 2) Q.R = 3) T.Q = 4) V.T =
5) P.R =

20
División de polinomios
Dividir polinomios es tan sencillo, como dividir cantidades enteras, sólo que un polinomio
es como un grupo de números enteros descompuestos en una adición de muchos
sumandos. Vamos a explicarlo por medio de un ejemplo:
Sabemos que el proceso de dividir consiste en: dadas dos cantidades “dividendo” y
“divisor”, se debe buscar otra cantidad llamada “cociente” que multiplicada por el “divisor”
nos resulte el “dividendo”.
Resolveremos la siguiente división de polinomios paso a paso:
(3x 2
) (
− 10 x 3 + 4 x 5 − x + 6 ÷ x 2 + 1 − 2 x )
Se ordenan los dos polinomios tomando en
cuenta los exponentes de la variable (x) en 4 x 5 + 0 x 4 − 10 x 3 + 3 x 2 − x + 6 x2 − 2x + 1
orden decreciente y completando con
coeficiente cero (0) la potencia faltante.
Se divide el primer término del polinomio 4 x 5 + 0 x 4 − 10 x 3 + 3 x 2 − x + 6 x2 − 2x + 1
dividendo entre el primer término del
divisor
Para efectuar esto se divide el coeficiente 4 x 5 + 0 x 4 − 10 x 3 + 3 x 2 − x + 6 x2 − 2x + 1
del dividendo entre el del divisor y con la
variable se aplica la regla de potencia de un 4x 3
cociente de igual base.
Este es el primer término del cociente
4 x5 4 x5
2
= 2
= 4 x( 5 − 2 ) = 4 x3
x 1x
Se multiplica el primer término del cociente 4 x 5 + 0 x 4 − 10 x 3 + 3 x 2 − x + 6 x 2 − 2 x + 1
por todos los términos del divisor, a estos
productos se les cambia el signo y se − 4 x5 + 8x 4 − 4 x3 4x 3
ordenan debajo del dividendo según el
exponente de la variable.
Estos productos se resta del dividendo 4 x 5 + 0 x 4 − 10 x 3 + 3 x 2 − x + 6 x 2 − 2 x + 1

− 4 x5 + 8x 4 − 4 x3 4x 3
8 x 4 − 14 x 3 + 3x 2 − x + 6

Se repite todo el procedimiento 4 x 5 + 0 x 4 − 10 x 3 + 3 x 2 − x + 6 x 2 − 2 x + 1


considerando que ahora el primer término
del nuevo dividendo es 8x4 − 4 x5 + 8x 4 − 4 x3 4 x3 + 8 x 2
8x 4 8 x 4 8 x 4 − 14 x 3 + 3x 2 − x + 6
2
= 2
= 8x( 4− 2) = 8x 2 − 8 x 4 + 16 x 3 − 8 x 2
x 1x
2 x3 − 5 x 2 − x + 6
Continuamos ahora dividiendo los demás términos

21
4 x 5 + 0 x 4 − 10 x 3 + 3 x 2 − x + 6 x 2 − 2 x + 1
− 4 x5 + 8x 4 − 4 x3 4 x3 + 8 x 2 + 2 x − 1
8 x 4 − 14 x 3 + 3x 2 − x + 6
− 8 x 4 + 16 x 3 − 8 x 2
2 x3 − 5 x 2 − x + 6
− 2 x3 + 4 x 2 − 2 x
− x 2 − 3x + 6
x2 − 2x + 1
− 5x + 7

El cociente de la división es : 4 x 3 + 8 x 2 + 2 x − 1

Y el residuo: − 5 x + 7 (como el grado de este residuo es inferior al del divisor, no se


puede continuar dividiendo por lo que la división es inexacta)

Ejercicios propuestos:
( )
1- 3 x 3 + 2 x 2 − 7 x + 2 ÷ ( x + 2 )

2- ( 9 x − x − 24 x − 3 x + 8 x ) ÷ ( x
4 5 3 2 2
−1)
3- (10 y − 20 y − y + 2 ) ÷ ( y − 2 )
8 6 2 2

 3 1 4 1 2 1 1 2 1
4-  x − x + x + x −  ÷  x − 
 2 3 2 2 3
5- ¿Cuál debe ser el valor de a, b y c para que se cumpla la siguiente igualdad?
(10 x a
) (
+ 50 x b − 5 x c ÷ 5 x 5 = 2 x 3 + 10 x 2 − 1)

LECTURA Nº 5: PRODUCTOS NOTABLES

Tomado con fines instruccionales de:


Santamaría, J. (2006). Productos notables.
Artículo no publicado (pp.1-8). Tinaquillo,
Estado Cojedes.

Al iniciarnos en nuestra aventura por el conocimiento de las matemáticas, lo primero a lo


que se hace referencia es al número, como clase, según lo plantean algunos, o como
conjunto, según otros. La cuestión es que el hombre y su inmensa necesidad de
organizarse en sociedades, poco a poco, fue implementando un lenguaje simbólico que le
sirvió de instrumento en las actividades cotidianas, tanto para comunicarse como para
demarcar y establecer normas de convivencia. Primero, se da cuenta que el medio
natural le ofrece una serie de herramientas para tal organización; comienza a utilizar las

22
piedras como mecanismo de conteo; luego, descubre que puede hacer marcas en los
árboles, en el suelo, en las paredes de las cavernas… y así llega, sin saber, a la intuición
de número.
El estudio de los números, o mejor dicho la fase de estructuración de los números y su
aplicación en otras ramas de la matemática, como la geometría, la aritmética y el álgebra,
no ha sido fácil. Desde mucho antes de Cristo, con Pitágoras de Samos, pasando por
Euclides, Al-Jwārizmī, Fermat, Descartes, Leibniz, entre otros; todos ellos fueron dándole
forma y sentido a todo ese conocimiento vago que desde tiempos remotos, babilonios y
egipcios aplicaban en su cotidianidad.
Por ejemplo, en la aritmética, que es la parte de la matemática que trata del arte o
habilidad para contar, sólo se utilizan números o cantidades conocidas que mediante
operaciones de adición, multiplicación y potenciación, de acuerdo con ciertas
propiedades ya existentes, es posible realizar todos los cálculos habidos y por haber. En
el álgebra, rama de la matemática que permite generalizar las aplicaciones aritméticas,
mediante el uso de cantidades desconocidas representadas por letras, también se vale
de las operaciones de adición, multiplicación y potenciación para tales aplicaciones. Y en
la geometría (del griego geō que significa 'tierra' y metrein 'medir'), rama de las
matemáticas que se encarga de las relaciones métricas del espacio y sus propiedades,
en su forma más elemental y no tan elemental; se vale del álgebra y la aritmética para
formalizar y sistematizar sus aplicaciones.
Dentro de todas estas operaciones elementales, como la adición, la multiplicación, la
potenciación, entre otras, aplicables en todas las ramas de las matemáticas
anteriormente mencionadas, a través de propiedades de composición bien definidas, se
derivan procedimientos que permiten simplificar con mayor facilidad las operaciones
indicadas. Procedimientos como el producto notable y la factorización son
herramientas muy prácticas para la agilización en la búsqueda de un resultado concreto.
Cuando se realiza un producto notable se está aplicando una multiplicación, pero se hace
de una forma directa reduciendo la operación a un mínimo de pasos posibles, por
ejemplo en aritmética no es muy frecuente encontrarse con un producto notable pero se
puede ejemplificar un ejercicio para hacer sencillas demostraciones, de la siguiente
manera:
(5 + 3) 2 = 5 2 + 2 ⋅ (5 ⋅ 3) + 3 2 = 25 + 30 + 9

Si se realiza la multiplicación aplicando la propiedad distributiva, que es el proceso


normal, el procedimiento se hace más largo; observa:

(5 + 3) 2 = (5 + 3) ⋅ (5 + 3) = 5 ⋅ 5 + 5 ⋅ 3 + 3 ⋅ 5 + 3 ⋅ 3 = 25 + 15 + 15 + 9

Ahora bien, si trabajamos dentro del álgebra, el mismo producto notable pudiese
aplicarse de la siguiente manera:

23
(3x + 5 y ) 2 = (3 x + 5 y ).( 3x + 5 y ) = (3x).( 3x ) + ( 3 x ).(5 y ) + (5 y ).( 3x ) + ( 5 y ).( 5 y )

(3x + 5 y ) 2 = (3 x) 2 + 2 ⋅ (3x ⋅ 5 y ) + (5 y ) 2 = 9 x 2 + 30 xy + 25 y 2
Al llevar este mismo procedimiento al campo de la geometría le daríamos el siguiente
enfoque:
Suponga un terreno de forma cuadrada, donde cada lado mide ” y − 4 ”, calcula el área del
terreno:
Para hallar el área de un cuadrado se multiplica lo que mide de ancho
por lo que mide de largo; así:
( y − 4) ⋅ ( y − 4) = ( y − 4) 2 = y 2 + 2 ⋅ ( y ) ⋅ ( −4) + ( −4) 2
y −4
= y 2 − 8 y + 16 , es el área del terreno
El producto notable es aquella multiplicación que se efectúa con expresiones algebraicas
de forma directa, aplicando una fórmula o procedimiento, de acuerdo a una situación
específica.
Veamos algunos casos específicos de productos notables.

EL CUADRADO DE UNA SUMA DE DOS TÉRMINOS


Ejemplo Nº 1
Supóngase que tenemos una región de forma cuadrada, cuyas dimensiones son las
siguientes: de largo y de ancho mide " x + 7" unidades.
Necesitamos conocer el área del cuadrado.
Sabemos que para calcular el área de un cuadrado,
x+7 sólo tenemos que multiplicar lo que mide de ancho por
lo que mide de largo, Es decir:
Área del Cuadrado = Largo ⋅ Ancho
x+7
Área = (Lado) 2
Entonces; Apliquemos la Fórmula:
Área = ( x + 7) ⋅ ( x + 7) = ( x + 7) 2

Ancho Largo Por Ley de Potenciación: a ⋅ a = a 2

Si aplicamos la propiedad distributiva nos quedaría:

(X + 7) . (X + 7) = X2 + X.7 + 7.X + 72 = X2 + 2 (7.X) + 72

Luego: Área = ( x + 7) 2
24
Desarrollemos esta potencia de la siguiente manera:

Doble

( x +7 ) 2
=x 2
+2 ⋅( x ) ⋅( 7 ) +( 7 ) 2

Primer Segundo Primer Segundo


Término Término Término Término

El resultado es un polinomio de tres Simplificando el resultado, tenemos


términos: “EL primer término al que:
cuadrado, más el doble del producto De esta manera obtenemos el área de
del primer término por el segundo, más la región cuadrada:
el segundo término al cuadrado” Área =

Ejemplo Nº 2:
Desarrollemos el Producto Notable: (5 + y ) 2

(5 +y ) 2 =(5) 2 +2 ⋅(5) ⋅ y +( y ) 2
El Doble del
Cuadrado del producto: del 1er Cuadrado del
1er Término término por el 2do 2do Término
Simplificando queda: término

(5 + y ) 2 = 25 + 10 y + y 2

Ejercicios propuestos:
5.1- (x + 7)2 5.2- (3X/2 + 4/9) 2 5.3- ( a/5 + 5) 2
5.4- (x2 + 3) 2 5.5- (xy + xz) 2 5.6- (Xa+1 + 1) 2
5.7- (a2 b + ac) 2 5.8- (2xy + y2 ) 2
5.9- En un club se desea crear una cancha para la práctica individual de tenis y se
dispone de una pared cuadrada de lado x. Los especialistas en ese deporte
solicitan que sea más grande, por lo que se le añadieron 3m a cada lado.
¿Cuál es el área de la nueva pared?

CUADRADO DE UNA DIFERENCIA DE DOS TÉRMINOS

Se resuelve de la misma forma que el caso del cuadrado de la suma de dos términos;
sólo que para desarrollar este caso hay que tomar en cuenta el signo de los términos.
Ejemplo Nº 3:
Doble

( x −3) 2 = x 2 +2 ⋅( x ) ⋅( −3) +( −3) 2

Primer Segundo Primer Segundo


Término Término Término Término
Simplificando:

25
( x − 3) 2 = x 2 − 6 x + 9 El cuadrado de una diferencia es igual a:
El cuadrado del primer término, menos el doble
producto del primero por el segundo, más el cuadrado
del segundo

Ejercicios propuestos:
5.10- (X - 5)2 5.11- (2X/3 - 1/5) 2 5.12- (a/3 - 3) 2
5.13- (X2 - 2) 2 5.14- (Xa-1 - 1) 2 5.15- (2xy - x2 ) 2

5.16- Si a2 + b2 = 13 y a . b = 6 ¿cuánto vale (a – b) 2?


5.17- Calcula los productos: a) (–x – a) 2
2
b) (x + a) ¿Qué relación existe entre ellos? ¿Por
qué?
5.18- Se necesita revestir un piso con cerámica, el cual tiene forma cuadrada de
lado x, pero la cantidad de cerámica sólo cubre una superficie también
cuadrada que tiene ¾ de metro menos por cada lado del área total.
¿Cuántos m2 de cerámica se compraron?
5.19- ¿Qué diferencia observas en estos ejercicios? : a) (x – a) 2
b) x2 - a2
después de resolverlos, ¿qué apreciación tienes al respecto?

EL PRODUCTO DE DOS BINOMIOS CON UN TÉRMINO COMÚN

Ejemplo Nº 4:
Tenemos una región de forma rectangular cuyas dimensiones ya conocemos:
Se necesita conocer el área de la región.

x−5 Sabemos que el área de un rectángulo se calcula


multiplicando lo que mide de largo por el ancho.
x+7
Entonces: Área = ( x + 7 ) ⋅ ( x − 5)

Largo Ancho

Desarrollarnos este producto de la siguiente manera:


( x +7 ) ⋅( x −5) =x 2 +x ⋅[7 +( −5)] +7 ⋅( −5)

Término Términos no Término Suma de Producto de


Común comunes común términos no términos no
comunes comunes

26
El resultado de este producto notable
es un trinomio: “El término común al
cuadrado más el producto del término Simplificando el resultado, queda:
común con la suma algebraica de los
términos no comunes más el producto
de los términos no comunes”. Trinomio

De esta manera se obtiene el área de la región rectangular:


Área = x 2 + 2 x − 35

Ejemplo Nº 5:
Desarrolla el producto: (3x − 9) ⋅ (3x + 2)

(3x −9 ) ⋅(3x +2) =(3x ) 2 +(3x ) ⋅( −9 +2) +( −9 ) ⋅ 2

Término Término no
Común comunes

Simplificando cada término:


(3 x ) 2 = (3 x ) ⋅ (3 x ) = 9 x 2
(3x ) ⋅ (−9 + 2) = (3 x) ⋅ (−7) = −21x
(−9) ⋅ 2 = −18
Luego:
(3x − 9) ⋅ (3x + 2) = 9 x 2 − 21x − 18
El producto de los términos no comunes
Producto del término común con la suma de los no comunes
El cuadrado del término común

Ejercicios propuestos:
5.19- (x2 + 6) . (x2 – 2) 5.20- (a3 + 1/5) . (a3 + 2/3)
5.21- (y – 3/5) . (y + 4) 5.22- (2x - 7) . (2x +2)
5.23- Si se cumple que (x + a) . (x + b) = x2 - 2x + 8 entonces ¿cuánto vale a + b?

5.24- ¿Para qué valores de la x se cumple que el producto de:


a) (x + 3) por
b) (x - 1) es igual a cero?
5.25- Si a un cuadrado cuya área mide x2 se le suma a un lado 9 cm y en el otro se
le resta 2cm, ¿cuál será el área de la nueva figura? x b
x
2.26- Calcule el área del siguiente rectángulo:
a
27
LA SUMA DE DOS TÉRMINOS POR SU DIFERENCIA:

Ejemplo Nº 6:
Se conocen las dimensiones de una región rectangular:
Largo = x + 6 y Ancho = x − 6
Tenemos que calcular el área respectiva:
Para hallar el área de un rectángulo aplicamos la
x−6 Fórmula: Área = Largo x Ancho.
o Área = base x Altura

x+6 Entonces, Área = ( x + 6) ⋅ ( x − 6)

Para desarrollar este producto procedemos de la siguiente forma:


( x +6) ⋅( x −6) =( x ) 2 −(6) 2
Suma Diferencia 2do Término al
1er Término al cuadrado
cuadrado
Simplificando el resultado:
x 2 − 36
El resultado de este producto notable es Luego:
un binomio: El área de la región rectangular es:
“El cuadrado del primer término menos el
x 2 − 62
cuadrado del segundo término”

Ejercicios propuestos:
5.27- (y – 3/5) . (y + 3/5) 5.28- (x2 + 6) . (x2 – 6) 5.29- (a3 + 1/5) . (a3 –
1/5)
5.30- (x/3 + 2/7) . (x/3 – 2/7) 5.31- (2x - 7) . (2x +7)
2
5.32- Si a un cuadrado cuya área mide x se le suma a un lado 5 m y en el otro se
le resta 5 m ¿cuál será el área de la figura que se originó? x a
5.33- Calcula el área de la figura sombreada:
x
a

EL CUBO DE UNA SUMA DE DOS TÉRMINOS:

Ejemplo Nº 6:
Se debe determinar el volumen de un tanque que tiene forma de cubo, conociendo
sus dimensiones:
Largo = x + 5, Ancho = x + 5 y Alto = x + 5
Para hallar el volumen de un cubo aplicamos la fórmula:
Volumen = Largo x Ancho x Alto

28
x+5
Como las tres medidas son iguales entonces
x+5 Volumen = (Lado)3

Entonces: Volumen = ( x + 5) ⋅ ( x + 5) ⋅ ( x + 5)

Por Ley de Potenciación: ( x + 5) ⋅ ( x + 5) ⋅ ( x + 5) = ( x + 5)3


Luego:
Volumen = ( x + 5) 3
Para desarrollar esta potencia procedemos así:
( x + 5) 3 = (x + 5)2 . (x + 5) esto por ley de potenciación y como sabemos calcular el
cuadrado de una suma
( x + 5) 3 = (x2 + 10.x + 25) . (x + 5)

( x + 5) 3 = x3 + 5.x2 + 10.x2 + 50.x + 25.x + 125 esto por multiplicación de polinomios

( x + 5) 3 = x3 + 15.x2 + 75.x + 125 por agrupación de términos semejantes

( x + 5) 3 = x3 + 3 . 5. x2 + 3. 52.x + 53

El resultado de este producto notable es un polinomio: “El cubo del primer término,
más el triple del producto del primer término al cuadrado, por el segundo término,
más el triple del producto del primer término por el cuadrado del segundo, más el
cubo del segundo término”.

Triple

( x +5)3 =(x )3 +3 ⋅( x ) 2 ⋅5 +3 ⋅( x ) ⋅(5) 2 +(5)3

Primer Segundo Primer Segundo


Término Término Término Término

Luego; simplificando cada término:


( x) 3 = x 3 , 3 ⋅ ( x ) 2 ⋅ (5) = 15 ⋅ x 2

(5) 3 = 5 ⋅ 5 ⋅ 5 = 125 , 3 ⋅ ( x ) ⋅ (5) 2 = 3 ⋅ x ⋅ 25 = 75 x


De esta manera tenemos que:
( x + 5) 3 = x 3 +15 x 2 + 75 x +125

29
Ejemplo Nº 7:
Desarrollar el producto notable: ( 2 x + 1) 3
Si aplicamos el procedimiento anterior; obtenemos:

El cubo del primer término (2x) 3

El triple del producto del primer término al 3 . (2x) 2 . 1


cuadrado por el segundo término

El triple del producto del primer término por 3 . 2x . 13


el cuadrado del segundo

El cubo del segundo término 13

Sumando estos términos


( 2 x +1) 3 = ( 2 x ) 3 + 3 ⋅ ( 2 x ) 2 ⋅ (1) + 3 ⋅ ( 2 x ) ⋅ (1) 2 + (1) 3

Simplificando cada término del resultado:


* (2x ) = (2x ) ⋅ (2x ) ⋅ (2x )
3 Luego, el polinomio se reduce a:
= 8x3
( 2 x + 1) 3 = 8 x 3 + 12 x + 6 x + 1
* 3 ⋅ ( 2 x ) ⋅ (1) = 3 ⋅ 4 x ⋅ 1
2 2

= 12x 2
* 3 ⋅ ( 2 x ) ⋅ (1) = 3 ⋅ 2 x ⋅ 1
2

= 6x
* (1) = 1 ⋅ 1 ⋅ 1 = 1
3

Ejercicios propuestos:
5.34- (x + 3)3 5.35- (3X/2 + 4/5) 3 5.36- ( y/3 + 3) 3
5.37- (x2 + 5) 3 5.38- (xy + xz) 3 5.39- (a2 b + ac) 3
5.40- (2xy + y2 ) 3
5.41- Si el volumen de un cubo es 27 cm3 ¿Cuál será el nuevo volumen si se
aumenta su arista en x unidades?

EL CUBO DE LA DIFERENCIA DE DOS TÉRMINOS.

Se desarrolla aplicando el mismo procedimiento de “el cubo de la suma de dos términos”,


sólo que en este caso se debe tomar en cuenta el signo de los términos.
Veamos esto en un ejemplo:

Ejemplo Nº 8:

30
Desarrolla el producto notable: ( y − 2) 3

( y −2) 3 = ( y ) 3 +3 ⋅( y ) 2 ⋅ ( −2) +3 ⋅ ( y ) ⋅ ( −2) 2 +( −2) 3

Primer Segundo
Término Término

Simplificando cada término en el resultado: Luego; Simplificado cada término el


* ( y) 3 = y 3 polinomio resultante es:
* 3 ⋅ ( y ) 2 ⋅ ( 2 ) = −6 y 2
( y − 2) 3 = y 3 − 6 y 2 + 12 y − 8
* 3 ⋅ ( y ) ⋅ ( −2) 2 = 3y ⋅ ( 4)
= 12 y
* ( −2) = ( −2) ⋅ ( −2) ⋅ ( −2)
3

= −8

En resumen, obtenemos como resultado: El cubo del primer término, menos el triple del
producto del cuadrado del primero por el segundo, más el triple del producto del primero
por el cuadrado del segundo, menos el cubo del segundo término.

Ejercicios propuestos:
5.42- (X – 1/2)3 5.43- (2X/3 - 1/5) 3 5.44- (a/3 - 3) 3
5.45- (X2 - 5) 3 5.46- (xy - xz) 3 5.47- (2xy - x2 ) 2

5.48- Compara los siguientes cubos a) (x - p) 3


b) (p - x) 3 ¿Son iguales? ¿Por qué?
5.49- Las cajas para embalaje de mercancía de una empresa tienen forma cúbica
con volumen de 125 cm3, con la finalidad de disminuir costos, la empresa
decide reducir el tamaño del envase restando x unidades (con x < 5) a la
arista del cubo original. ¿Qué fórmula permite conocer el volumen del nuevo
envase?
5.50- Si a = b + 3 ¿cuánto vale (a – b) 3 ?
5.51- Simplifica las siguientes operaciones:
a) 3 ⋅ ( 2 x + 1) − ( 4 x + 1) ⋅ ( 4 x − 1) =
2

[
b) 2 ⋅ ( 7 x + 3) ⋅ (7 x − 11) − 4 ⋅ ( x − 9) =
2
]
c) 2 ⋅ (3x + 1) − ( x − 6) =
3 3

5.52- Halla la suma de: el doble del cuadrado de la diferencia entre X y 2, con el
triple del producto de la suma de X y 1 por su diferencia.

31
LECTURA Nº 6: LA FACTORIZACIÓN COMO
HERRAMIENTA DE SIMPLIFICACIÓN

Tomado con fines instruccionales de:

Santamaría, J. (2006). La factorización como


herramienta de simplificación. Artículo no
publicado (pp.1-2). Tinaquillo, Estado Cojedes.

El procedimiento contrario al producto notable es la factorización, el cual es un proceso


que consiste en transformar una expresión algebraica en un producto o multiplicación.
Cuando un número o cualquier otra expresión no pueden descomponerse en factores, se
dice que es un número primo.
En las operaciones aritméticas y algebraicas se utiliza mucho el procedimiento de la
factorización como herramienta, para simplificar y resolver los ejercicios con menor
dificultad y mayor rapidez.
Por ejemplo:
Aritméticamente:
Observa que hay una suma de fracciones;
3 15 9 3 3⋅ 5 3⋅ 3 1 3
+ − = + − = +5− tanto en el numerador como en el
12 3 15 3 ⋅ 4 3 3 ⋅ 5 4 5 denominador de cada fracción se hizo una
descomposición en factores con aquellos
números que no son primos, ejemplo: 12 =
3 · 4, 15 = 3 · 5 y 9 = 3 · 3
3 15 9 3 3⋅ 5 3⋅ 3 1 3 Luego se cancelaron aquellos factores
+ − = + − = +5−
12 3 15 3 ⋅ 4 3 3 ⋅ 5 4 5 iguales en el numerador y denominador de
cada fracción, simplificándose cada
término.
En el álgebra:

x+2 x2 − 5x x+2 x( x − 5) Aquí tenemos otra suma de


+ = +
x + 4 x + 4 x − 25 ( x + 2)( x + 2) ( x + 5)( x − 5)
2 2 fracciones, pero no es aritmética
como la anterior.
Se hizo una descomposición en
factores en el numerador y
denominador "dex +cada
2" fracción. La
expresión no se pudo
1 x descomponer por ser un polinomio
= + primo. Luego, se simplificó cada
( x + 2) ( x + 5) fracción cancelando factores iguales
en el numerador y denominador.

Cada fracción algebraica está compuesta por expresiones llamadas polinomios, que para
factorizarlos se debe tener en cuenta algunas reglas, un ejemplo de ello es la expresión
" x 2 + 4 x + 4" , que representa un trinomio de cuadrado perfecto. Para factorizar este tipo
32
de expresión primero se debe estar familiarizado con ella, pues existen muchos casos de
factorización para ciertos tipos de polinomios.

LECTURA Nº 7: ¿CÓMO COMPLETAR CUADRADOS?

Tomado con fines instruccionales de:


Suárez, E. y Cepeda, D. (2003). Matemática
de Educación Básica. Editorial Santillana,
S.A. (p. 149). Caracas, Venezuela:

Fueron primero los griegos, y luego los árabes, los que utilizaron métodos geométricos
para dar con la solución de muchos de los problemas que hoy en día se resuelven
mediante la simbología algebraica. Por ejemplo, Mohammed al-Khowarizmi propuso,
hacia el año 825, un método geométrico para obtener una solución positiva de una
ecuación cuadrática.
De acuerdo con lo que él proponía, para resolver la ecuación x 2 + 8 x = 33 , se
siguen los siguientes pasos:
Suponemos que x 2 + 8x es una suma de áreas, la cual nos da 33 unidades

cuadradas, observemos el gráfico:

El cuadrado tiene lados de medidas x unidades, para hallar su área


multiplicamos lo que mide de ancho por lo que mide de largo. Así:
x Largo . ancho = x . x = x2

x
Luego,
Observa que se ha construido rectángulos a cada lado del cuadrado,
2 cuyos lados miden “x” y “2” unidades, respectivamente (esta medida
2 x “2” se obtiene de dividir “8”, que es el coeficiente del término lineal
8x, entre el número de rectángulos).
x x Al calcular el área de uno de estos rectángulos resulta:
x 2
2 Largo . Ancho = 2 . x

Entonces, al construir cuatro rectángulos, se forma un área entre todos ello que está
representada por:

2 x + 2 x + 2 x + 2 x = 8x
El área total de los rectángulos, más el área del cuadrado resulta x 2 + 8 x = 33
2
2
Ahora, se construyen cuadrados pequeños en cada esquina de la
figura para completar el cuadrado mayor.
Como podrás darte cuenta cada cuadrito tiene lado igual a 2
2
unidades, siendo el área 2 · 2 = 4 unidades cuadradas.
2
33
Entonces, entre los cuatro cuadritos se tiene un área igual a 4 ∙ 4 = 16 unidades
cuadradas, lo que indica que el cuadrado mayor tiene un área de: 33 + 16 = 49
Luego,
Tenemos un cuadrado cuyos lados miden (2 + x + 2) = x + 4
2 por lo que el área sería:
Largo . ancho = (x + 4).(x + 4) = (x + 4)2
x Pero ya se conoce el área total que es 49 unidades cuadradas
Entonces:
(x + 4)2 = 49 donde despejando el cuadrado nos queda:
2
x + 4 = 49
2 x 2 x+4=7
x =7–4
x=3

Entonces, volviendo al problema original, el área del cuadrado de


lado x es igual a: 3 . 3 = 9 unidades cuadradas x
x
LECTURA Nº 8: MÉTODOS DE FACTORIZACIÓN

Tomado con fines instruccionales de:


Ochoa, A. (2007). Métodos de Factorización. Unefa.
Artículo no publicado (pp.1-6). Caracas. Venezuela.

La operación de descomponer en factores los productos notables, también se llama


“Factorización”. Es el proceso inverso al desarrollo de los productos notables.
Para factorizar polinomios hay varios métodos:

FACTOR COMÚN
Consiste en transformar la expresión dada en un producto, donde uno de los factores es
común entre los términos y el otro se obtiene al dividir cada término de la expresión
original entre el factor común.
Ejemplo Nº 1: 12x + 3

3 ⋅ 4.x + 3 Descomponemos el número 12 en dos factores


y observamos que el 3 es común en los dos
términos.
3 Multiplicamos y dividimos toda la expresión
( 3.4.x + 3) = por el factor común
3
 3.4.x 3  Efectuamos el cociente de cada término entre
3. + = en factor común
 3 3
3.( 4 x + 1) = Esta es la expresión ya factorizada

34
Cuando nos piden sacar factor común o simplemente factorizar y hay coeficientes con
factores comunes, se saca el máximo común divisor de dichos coeficientes.
36 x 2 −12 x 3 +18 x
Ejemplo Nº 2: factorizar el polinomio

Ordenamos y calculamos el máximo común divisor entre


− 12 x 3 + 36 x 2 + 18 x los coeficientes de cada término,
mcd(36,12,18) = 6
Como la variable x es común en los tres términos,
− 12 x 3 + 36 x 2 + 18 x multiplicamos el mcd por la x elevada a la menor
potencia que aparezca. En este caso es elevada a la 1 (6x)
6x
6x
(
. − 12 x 3 + 36 x 2 + 18 x ) Multiplicamos y dividimos toda la expresión por este
factor común

 12 x 3 36 x 2 18 x  Efectuamos el cociente de cada término entre el factor


6 x. − + +  común
 6x 6x 6 x 
Resolviendo cada cociente:
(
6 x. − 2 x 2 + 6 x + 3 ) - Se dividen los coeficientes, y
- Se aplica la ley de cociente de potencias de igual base
(se copia la base y se restan los exponentes) y así se
obtiene la expresión factorizada por factor común
Ahora extraeremos factores comunes diferentes por agrupación de términos.
Ejemplo Nº 3: factorizar 3 x 2 − 6 xy + 4 x − 8 y

(3x 2
)
− 6 xy + ( 4 x − 8 y ) Formamos dos grupos considerando que los dos
primeros términos son divisibles entre 3x y los
dos últimos entre 4
3x 2
3x
( 4
)
3 x − 6 xy + ( 4 x − 8 y )
4
Multiplicamos y dividimos las dos expresiones
por estos factores comunes

 3x 2
6 xy   4 x 8 y  Simplificando
3 x −  + 4 − 
 3x 3 x   4 4 
3 x.( x − 2 y ) + 4( x − 2 y ) Observa que surgió un nuevo factor común entre
los dos términos.
( x − 2 y ) [ 3x.( x − 2 y ) + 4( x − 2 y ) ] Se procede a multiplicar y dividir por el nuevo
( x − 2 y) factor común

( x − 2 y )  3x.( x − 2 y ) + 4( x − 2 y ) 
Simplificando

 ( x − 2 y) ( x − 2 y) 
( x − 2 y )( 3x + 4) Obtenemos la expresión ya factorizada

DIFERENCIA DE CUADRADOS
Este caso se basa en la fórmula:
a2 – b2 = (a + b) . (a – b)

35
Ejemplo Nº 4: factorizar x2 – 9

x 2 − 9 = x 2 − 32 Expresamos todos los términos en cuadrados


x 2 − 9 = ( x + 3).( x − 3) Tomando en cuenta que la factorización es el
procedimiento inverso a producto notable y como
( a + b ).( a − b ) = a 2 − b 2
Ejemplo Nº 5: factorizar x4 – 16

( ) 2
x 4 − 16 = x 2 − 4 2 Expresamos todos los términos en cuadrados

( )(
x 4 − 16 = x 2 + 4 . x 2 − 4 ) Tomando en cuenta que la factorización es el
procedimiento inverso a producto notable y como:
( a + b ).( a − b ) = a 2 − b 2
( )
x 4 − 16 = x 2 + 4 .( x + 2 ).( x − 2 ) Como el segundo factor también es una diferencia
de cuadrados, se procede a factorizarlo:
x 2 − 4 = x 2 − 22

TRINOMIO
Se pueden conseguir tres casos:
Trinomio de la forma x2 + ax + b:
La fórmula general viene dada por:
x2 + ax + b y al factorizarlo queda expresada como
(x + n).(x + m) donde n.m = b y n + m = a
Ejemplo Nº 6:
x 2 − 7 x + 12
-3-4=-7 Buscamos dos cantidades, tales que su
(-3).(-4) = 12 producto sea 12, estás deben tener el
mismo signo para que el producto sea
positivo, y para que su suma sea -7, deben
ser los dos negativos.
x 2 − 7 x + 12 = x 2 + ( − 3 − 4 ) x + ( − 3).(−4 ) Se sustituyen los coeficientes, una por una
adición y la otra por una multiplicación
x 2 + 10 x + 24 = ( x − 3).( x − 4 ) Aplicando la fórmula general
Ejemplo Nº 7:
x 2 − 7 x + 12
6 + 4 = 10 Buscamos dos cantidades, tales que su suma
6 . 4 = 24 sea 10 y su producto sea 24
x 2 + 10 x + 24 = x 2 + ( 6 + 4 ) x + ( 6.4 ) Se sustituyen los coeficientes, una por una
adición y la otra por una multiplicación.
x 2 + 10 x + 24 = ( x + 6 ).( x + 4 ) Aplicando la fórmula general.

36
Ejemplo Nº 8:
x 2 + 15 x − 100
20 + (-5) = 15 Buscamos dos cantidades tales que su suma sea 15 y su
20 . (-5) = -100 producto sea -100. Para que el producto sea negativo
deben ser de signos diferentes.
x 2 + 10 x + 24 = x 2 + ( 20 + ( − 5) ) x + ( 20.( − 5) ) Se sustituyen los coeficientes, una por
una adición y la otra por una
multiplicación.
x + 10 x + 24 = ( x + 20 ).( x − 5)
2
Aplicando la fórmula general
3.2 Trinomio cuadrado perfecto
Se basa en las siguientes fórmulas

(a +b )2 =a 2 +2 ab +b 2 (a −b )2 =a 2 −2 ab +b 2
y
Analizamos el procedimiento mediante el ejemplo Nº 9:
x 2 + 25 + 10 x
x 2 + 10 x + 25 Verificamos si dos de los términos se pueden
2
X ya está en forma de cuadrado expresar en forma de cuadrado.
25 = 52
10 x = 2( x.5) También verificamos si el término restante se
puede expresar como el doble producto de las
bases de los cuadrados.
x 2 + 10 x + 25 = ( x + 5)
2
Al cumplir las condiciones, se pasa a factorizarlo
según la fórmula.

Ejemplo Nº 10:

4 x 2 − 12 x + 9 =
4 x 2 − 12 x + 9 = Verificamos si dos de los términos se
4x2 = ( 2x)
2
pueden expresar en forma de cuadrado.
9 = ( − 3)
2

− 12 x = 2.( 2 x ).( − 3) También verificamos si el término restante


se puede expresar como el doble producto
de las bases de los cuadrados.
4 x 2 − 12 x + 9 = ( 2 x ) + 2.( 2 x.( − 3) ) + 32
2
Expresamos el trinomio en cuadrados y
productos.
4 x 2 − 12 x + 9 = ( 2 x − 3)
2
Factorizamos aplicando la fórmula.
ax 2 +bx +c
Trinomio de segundo grado ( )

Cuando no se cumplen las condiciones de los dos casos anteriores.

37
Para la factorización de este caso se procede de la siguiente manera:

ax 2 + bx + c = 0 Se iguala toda la expresión a cero (0).


Se calculan los dos valores de x, utilizando la
− b ± b − 4ac
2
x= ecuación cuadrática.
2a
ax 2 + bx + c = a ( x − x1 ).( x − x2 ) Se aplica la fórmula general.

Ejemplo Nº 11:
2 x 2 +5 x −3
Factorizar el polinomio

2 x2 + 5x − 3 = 0 Igualamos a cero y determinamos los valores de a,


a=2 b=5 c = -3 b y c.
− 5 ± 5 − 4.2.( − 3)
2 Sustituimos los valores de a, b y c en la ecuación
x= cuadrática
2.2
Resolviendo lo que está dentro de la raíz:
− 5 ± 25 + 24 52 = 25
x=
2.2 -4 . 2 . (-3) = -8 . (-3) = + 24

− 5 ± 49
x=
2.2
Extraemos la cantidad subradical por ser un
−5±7 cuadrado perfecto.
x=
4
Obtenemos dos valores de la x uno sumando 7 y el
−5+7 2 1 otro restándolo.
x1 = = =
4 4 2 Así obtenemos:
1
x1 = x2 = −3
− 5 − 7 − 12 2
x2 = = = −3
4 4

Reemplazamos los valores en la fórmula general.


 1
2 x 2 + 5 x − 3 = 2 x − .( x + 3) Recuerda que x-(-3) = x + 3
 2

Regla de Ruffini
38
Se aplica para cualquier polinomio que tiene raíces enteras; es decir, encontrar valores
de x (números enteros) que al sustituirlos en el polinomio nos da cero.
ax 4 +bx 3 +cx 2 +dx +e
Por ejemplo, si un polinomio de cuarto grado , tiene cuatro raíces
x x x x
enteras, , , y se factoriza así:
1 2 3 4

ax 4 +bx 3 +cx 2 +dx +e =a (x −x1 )(x −x 2 )(x −x 3 )(x −x 4 )

Pero ¿cómo se aplica la regla de Ruffini para obtener las raíces?


x4 −4x 3 −x2 +16 x −12
Ejemplo Nº 12: Factorizar
Se aplica la regla de Ruffini, probando los divisores del término independiente, en este
caso de 12, o sea que se prueba con 1, -1, 2, -2, 3, -3, 4, -4, 6, -6, 12 y –12
Probemos con uno (1)
x4 − 4 x3 − x2 + 16 x − 12
1 -4 -1 16 -12 Se copian los coeficientes del polinomio.
Escribimos el número seleccionado a la
1 derecha (a este lo llamaremos raíz).
1 Se copia el primer coeficiente debajo de él
mismo.

1 -4 -1 16 -12 Se multiplica la raíz por el primer


1 coeficiente que se bajó y el producto se
1 copia en la segunda fila debajo del segundo
1 -3 coeficiente. Luego se efectúa la suma
algebraica de las dos cantidades ubicadas
en la columnas donde se colocó el
producto.

1 -4 -1 16 -12 Se multiplica la raíz por el resultado de la


1 suma algebraica realizada y este producto
1 -3 se copia en la segunda fila debajo del tercer
1 -3 -4 coeficiente. Luego se efectúa la suma
algebraica de las dos cantidades ubicadas
en las columnas donde se colocó el
producto.

1 -4 -1 16 -12 Se vuelve a multiplicar y sumar el producto


1 con el siguiente coeficiente.
1 -3 -4
1 -3 -4 12

1 -4 -1 16 -12 Se efectúa el último producto y la última


1 suma. Como el resultado final es cero (o),
1 -3 -4 12 esto nos indica que el 1 sí es una raíz del
1 -3 -4 12 0 polinomio y nos sirve para factorizar.
(x – 1) . ( x
3
− 3x 2 − 4 x + 12 ) Hasta ahora tenemos un producto como se
observa al utilizar los nuevos coeficientes
obtenidos.

Si hubiera dado distinto de cero habría que seguir probando los demás divisores de 12.

39
De hecho ya hemos factorizado el polinomio, pero el segundo factor de tercer grado
debemos intentar seguir factorizándolo.
Probando ahora por 2 y aplicando otra vez la regla queda:

1 -4 -1 16 -12
1 1 -3 -4 12
1 -3 -4 12 0
2 2 -2 -12
1 -1 -6 0
Así hemos conseguido la segunda raíz, por lo que el polinomio va quedando factorizado
de la siguiente manera:
( x − 1).( x − 2).( x 2 − x − 6)
Ahora seguimos aplicando la regla para encontrar las otras raíces.

1 -4 -1 16 -12
1 1 -3 -4 12
1 -3 -4 12 0
2 2 -2 -12
1 -1 -6 0
-2 -2 6
1 -3 0
La nueva raíz en -2 y el último cociente se toma con la raíz -3
La factorización final es:
x4 −4x 3 −x2 +16 x −12 (x −1)(x −2 )(x +2 )(x −3)
=
Si en las sucesivas pruebas no encontramos ningún resto cero, quiere decir que el
polinomio no se puede factorizar dentro de los números reales.
RESUMIENDO:
Según como sea el polinomio hay métodos que se pueden aplicar y otros que no. Se
aconseja que se intenten aplicar los cinco métodos sucesivamente, es decir, en primer
lugar se puede extraer el factor común, y luego se pueden seguir aplicando otros de los
métodos.
Ejercicios propuestos:

40
Factoriza:
6.1- x2 + 2x + 3 6.2- x2 − a2 + x − a2 x
6.3- 3 x 5 − 48 x 6.4- 4 x12 + 12 x 6 + 9
6.5- x 3 − 12 x 2 + 41x − 30 6.6- 3 xm 2 − x + 3m 2 − 1
6.7- 3 x 2 + 15 x + 18 6.8- 3 x 3 + 3x 2 + 3x + 3
x 2 xy y 2 a 2 b4
6.9- + + 6.10- −
4 3 9 100 9
Calcula el valor de k en:

6.11- P( x ) = −2 x 4 − 6 x 3 + 5 x − k si P( − 2) = 35

1 2 1
6.12- P( x ) = 8x 4 − x + 12 x + k si P  = 125
4 2

6.13-Si el volumen de un paralelogramo viene dado por la fórmula: V = x 3 + 5 x 2 + 6 x .


¿Cuáles podrían ser las medidas de las aristas (largo, ancho y altura)?

(
6.14-¿Para qué valor de n se cumple que x n − x = x x 2 − 1 ? )
6.15-¿De cuántas maneras podemos factorizar el número 64?

41
UNIDAD 2
VALOR ABSOLUTO, INECUACIONES
LECTURA Nº 9: NUMERACIÓN ANTIGUA EGIPCIA

Tomado con fines instruccionales de:

Perelman. (2002). Aritmética recreativa.


Traducida por Barros P. Editorial URSS.
Antofagasta URSS.

La numeración egipcia es una de las más antiguas, data aproximadamente de hace


7000 años, más de tres mil antes de nuestra era. En el transcurso de los tres primeros
milenios estos símbolos sufrieron cambios insignificantes, fijemos nuestra atención en la
forma que los egipcios representaban los signos numéricos y cómo los escribían.
En la numeración egipcia, existían signos especiales (jeroglíficos) para los números: uno,
diez, cien, mil, diez mil, cien mil, un millón, cada uno de ellos está representado en las
figuras que observamos a continuación:

Estos signos especiales (jeroglíficos) eran utilizados por los antiguos egipcios para la
notación de los números.
Para representar, por ejemplo, el número entero 23145, era suficiente escribir en serie
dos jeroglíficos de diez mil luego tres jeroglíficos de mil, uno de cien, cuatro de diez y
cinco jeroglíficos para las unidades.

Estos símbolos en la escritura, no podían aparecer más de nueve veces en cada número.
Este ejemplo es suficiente para aprender a escribir los números tal y como los
representaban los antiguos egipcios. El sistema egipcio de numeración es muy simple y
primitivo, no hay signo alguno para el cero, es un sistema decimal puro puesto que en
la representación de los números enteros, se emplea el principio decimal conforme al
orden clase. Se puede notar que cada signo numérico representa solamente un número.
Así, por ejemplo, el signo para las decenas denota solamente diez unidades y no diez
decenas o diez centenas, lo que evidencia por qué el sistema de numeración egipcio no
era posicional.

42
LECTURA Nº 10 EL VALOR ABSOLUTO Y LOS
NÚMEROS REALES
Tomado con fines instruccionales de:

Santamaría, J. (2007). El Valor Absoluto y


los Números Reales. Artículo no publicado
(pp.1-2). Tinaquillo, Estado Cojedes.

El valor absoluto de un número real, se puede definir como la distancia que existe entre
dos posiciones simétricamente iguales, partiendo de un mismo punto de referencia; esto
se puede ilustrar como sigue, tomemos una recta y la enumeramos tanto con valores
enteros negativos como positivos, y luego tomando el cero como punto de referencia,
establecemos una distancia tanto a la izquierda como a la derecha:

?
Teléfono Público Teléfono Público

-4 -3 -2 -1 0 1 2 3 4

Vamos a suponer, que un individuo se encuentra en una parada de autobús y decide


hacer una llamada telefónica de urgencia desde un teléfono público. El teléfono más
cercano se encuentra a cierta distancia a la derecha de donde él está, pero hacia la
izquierda esté otro teléfono exactamente a la misma distancia. La pregunta es: ¿a cuál
teléfono se dirigirá? ¿cuál le queda más cerca?
Según el grafico anterior, podemos deducir que el individuo se puede dirigir a cualquiera
de los dos teléfonos, pues ambos están a igual distancia de donde él se encuentra. Es
decir, el valor absoluto de los “+4” y “-4”, nos da el mismo resultado “4”. Esto se puede
representar de la siguiente forma:
+4 =4 y − 4 = −( − 4 ) = 4 entonces ⇒ + 4 = − 4
El valor absoluto de una expresión numérica se suele representar entre barras. De esta
situación podemos deducir que el valor absoluto de un número real cualquiera (positivo o
negativo) es el número siempre positivo. Ahora bien, definamos esto en términos
matemáticos:

 x , si x > 0

x = 0 , si x = 0 siendo “ x ” cualquier número real.
− x , si x < 0

El valor absoluto no sólo se aplica a cantidades conocidas, sino también a expresiones


desconocidas o algebraicas, para ello es necesario conocer las propiedades que lo
conforman como estructura matemática. Entre esas propiedades se encuentra las
siguientes:

43
1.- El valor absoluto de una adición de dos números reales cualesquiera, es menor o
igual a la suma de los valores absolutos de cada número real. En lenguaje
matemático esto es:
x+ y ≤ x + y

siendo “ x ” e “ y ” dos números reales cualesquiera ( x, y ∈ R)

Ejemplo Nº 1: − 9 + 5 ≤ − 9 + 5

⇒ −4 ≤9+5

⇒ 4 ≤ 14 se comprueba la desigualdad.

2.- El valor absoluto de una multiplicación de dos o más números reales, es igual a la
multiplicación de los valores absolutos de cada número real. En lenguaje matemático
sería:
x ⋅ y = x ⋅ y , x, y ∈ R

Ejemplo Nº 2: (−9) ⋅ 5 = − 9 ⋅ 5

− 45 = 9 ⋅ 5

45 = 45
Probemos ahora con dos números enteros negativos:

Ejemplo Nº 3: (−1) ⋅ (−7) = − 1 ⋅ − 7

7 = 1. 7

7=7

3.- El valor absoluto de una división de dos números reales, es igual a la división de los
valores absolutos de cada número real. En lenguaje matemático es:
x x
= , siendo x, y ∈ R con y ≠ 0
y y

21 21
Ejemplo Nº 4: =
−3 −3

21
⇒ −7 =
3
⇒7=7
Ejercicios propuestos:

10.1) ¿A qué es igual 9 ? 10.2) Calcula 1 − π + − 3

44
10.3) ¿A qué es igual (x 2
)
−9 ? 10.4) ¿Cuánto vale x si x + 2 = 1 − x ?

10.5) Si − 2 x + 5 = 0 entonces x = 2( x + 2)
10.6) Si 3 − = 3 entonces x =
5

LECTURA Nº 11: LOS INTERVALOS Y EL CALENDARIO


Tomado con fines instruccionales de:
Suárez, E. y Cepeda, D. (2003).
Matemáticas de Educación Básica.
Editorial Santillana, S.A. (p.110). Caracas,

Los intervalos se han utilizado prácticamente desde los comienzos de nuestra civilización,
el hombre mediante la observación de los fenómenos naturales, comenzó a registrar el
tiempo a través de marcas en los árboles o en sus cuevas. Con el tiempo, se estableció
el año de 360 días, dividido en 12 meses y 4 estaciones; pero las civilizaciones que
usaban el calendario señalado se percataron de que este cálculo no era exacto y tenían
que agregar días para predecir el período de siembra y cosecha. Fue en el año 45 AC.
cuando el emperador romano Julio César fijó la duración del año en 365 días y ordenó se
acumularan 6 horas por año, y que cada cuatrienio (4 años) se aplicará un día más, lo
cual debía llevarse a cabo en el mes de febrero; así surgió el año bisiesto.
Aunque el cálculo de Julio César fue muy aproximado, cometió un error, pues al año solar
no le sobraban 6 horas, sino 5 horas, 48 minutos y 46 segundos. Esta pequeña
diferencia no fue grave al principio, pero hacia el siglo XVI (casi 600 años después) ya se
había producido una diferencia tan grande y un desplazamiento de las estaciones, que a
causa de ello, el Papa astrónomo Gregorio XIII, en el año de 1582 determinó adelantar al
calendario 19 días para actualizarlo; éste fue más preciso, apenas tiene un error de 1 día,
4 horas y 48 minutos en 4000 mil años.
El calendario se origina, por la necesidad de registrar el tiempo en función de los
intereses de aquella época. Cabe destacar que el término intervalo, utilizado con
frecuencia en matemática, es aplicable para fijar parámetros en los registros del tiempo,
cuando se hace referencia a ciertos períodos o momentos que ocurrieron, ocurren o
están por ocurrir, por ejemplo: milenios, siglos, décadas, años, meses, días, horas,
minutos, segundos, entre otros.

45
LECTURA Nº 12: INECUACIÓN CONTRA ECUACIÓN

Tomado con fines instruccionales de:


Santamaría, J. (2007). Inecuación contra ecuación.
Artículo no publicado (pp.1-2). Tinaquillo, Estado Cojedes.

Inecuación Ecuación
Es una desigualdad entre dos expresiones Es una igualdad entre dos expresiones
algebraicas, en la cual aparecen algebraicas, en la cual aparecen
constantes y una o varias variables constantes y una o varias variables
desconocidas llamadas incógnitas. desconocidas llamadas incógnitas.

Ejemplos: x − 2 ≤ 5 ; x 2 + y 2 ≥ −1 ; 1
Ejemplos: x − 2 = 5 ; x2 − = 1;
2
x< y ; log( 2 x + a ) > 0 1
sen( x) = ; x − y = −6 .
2
Si a ambos miembros de una inecuación se Si a ambos miembros de una ecuación se
les suma o se les resta un mismo número, les suma o se les resta un mismo
la inecuación no se altera. número, la ecuación no se altera.
Si se multiplican o dividen ambos miembros Si se multiplican o dividen ambos
de una inecuación por un mismo número miembros de una ecuación por un mismo
no nulo, resulta que la inecuación: número positivo o negativo no nulo, la
• No se altera, si el número es positivo. ecuación no se altera.
• Cambia el signo de desigualdad, si
el número es negativo.
Un punto (de la recta, plano,...) es una Un punto (de la recta, plano,...) es una
solución de una inecuación, si al sustituir solución de una ecuación, si al sustituir
las variables por los correspondientes las variables por los correspondientes
valores de las coordenadas del punto, la valores de las coordenadas del punto, la
desigualdad numérica resultante es igualdad numérica resultante es
verdadera. verdadera.

46
LECTURA Nº 13: CONOCIENDO LAS INECUACIONES
Tomado con fines instruccionales de:
Santamaría, J. (2007). Conociendo las
inecuaciones. Artículo no publicado (pp.1-
2). Tinaquillo, Estado Cojedes.

Las inecuaciones forman desigualdades entre dos o más expresiones algebraicas, donde
cada una de estas expresiones pertenecen a miembros de la desigualdad, una
inecuación encuentra solución en el conjunto de todos los valores de las incógnitas que
verifican la desigualdad, dicho conjunto recibe el nombre de “Conjunto Solución” y se
representa generalmente con la letra “S”. Para realizar cualquier operación relacionada
con inecuaciones, es necesario conocer las propiedades que rigen las desigualdades.
Veamos cada caso:
1) Si sumamos o restamos un mismo número a ambos miembros de una desigualdad,
se obtiene otra desigualdad del mismo sentido de la primera.

Si a > b ⇒ a + c > b + c Si a < b ⇒ a − c < b − c

Ejemplo Nº 1: 16 > 8 ahora le sumamos 2 a cada término


16 + 2 > 8 + 2 se mantiene la desigualdad con el resultado
18 > 10
Ejemplo Nº 2:
-5 < 8 ahora le sumamos -3 a cada término
- 5 + (-3) < 8 + (-3) se mantiene la desigualdad con el resultado
-8 < 5

2) Si los dos miembros de una desigualdad se multiplican o dividen por un mismo


número positivo, la desigualdad no cambia de sentido.

si a > b y c > 0 ⇒ a.c > b.c


si a < b y c > 0 ⇒ a.c < b.c
1 1
si a < b y c > 0 ⇒ a. < b.
c c

Los intervalos denotan el conjunto solución de una inecuación. Es recomendable y casi


necesario graficar dichos intervalos para visualizar el conjunto solución.
A continuación podemos observar una tabla que muestra los diferentes intervalos
unidimensionales:

47
INTERVALOS REPRESENTACION NOMBRE

[ a, b ] = { x ∈ R / a ≤ x ≤ b } Intervalo Cerrado
a b

( a, b ) = { x ∈ R / a < x < b } Intervalo Abierto


a b
Intervalo Semiabierto en
( a, b] = { x ∈ R / a < x ≤ b}
a b a
Intervalo Semiabierto en
[ a, b ) = { x ∈ R / a ≤ x < b}
a b b
///////////////////////////////////// Abierto en “a” hasta
( a,+∞ ) = { x ∈ R / x > a} infinitos valores
a +∞ positivos
[ a,+∞ ) = { x ∈ R / x ≥ a} ///////////////////////////////////// Cerrado en “a” hasta
infinitos valores
a +∞ positivos
( − ∞, a ) = { x ∈ R / x < a} ////////////////////////////////////// Abierto en “a” hasta
infinitos valores
−∞ a negativos
( − ∞, a ] = { x ∈ R / x ≤ a} ////////////////////////////////////// Cerrado en “a” hasta
infinitos valores
−∞ a negativos
Al resolver inecuaciones de 1º grado en una variable, se debe tener presente todos los
procesos antes vistos.
Resuelve las siguientes inecuaciones:
Ejemplo Nº 1: 3 x − 7 < 5
Solución:
a) Se suma el opuesto de -7 a ambos lados de la desigualdad.
a) 3 x − 7 + 7 < 5 + 7
b) Se efectúan las sumas algebraicas (-7+7=0) y (5+7=12)
b) 3 x < 12
c) Se multiplica por el inverso multiplicativo de “3” que es “
1 1 1
c) 3x  < 12  ” a ambos lados de la desigualdad.
3 3 3
3  12  d) Se simplifican las expresiones
d)   x <  
3 3 e) Se encuentra el intervalo solución y se procede a graficar.
d) x < 4

S= ( − ∞,4 ) ////////////////////////////////////////////////////

−∞ 4
Al sumar el opuesto a cada lado de una desigualdad, es lo mismo que se
hace cuando decimos que “si está sumando lo pasamos al otro extremo
restando y viceversa” 3x – 7 < 5 queda 3x < 5 + 7
Recordar:
Cuando indicamos que se multiplica por el inverso multiplicativo, es lo
mismo que se hace cuando decimos que “si está multiplicando pasa 48
dividiendo y viceversa”
Ejemplo Nº 2: − 9 ≤ 2 x − 5 ≤ 9
Solución: a) Sumamos el opuesto aditivo de -5 en todos los
a) − 9 + 5 ≤ 2 x − 5 + 5 ≤ 9 + 5 miembros de la desigualdad.
b) Se efectúan las sumas algebraicas.
b) − 4 ≤ 2 x ≤ 14 c) Se multiplica por el inverso multiplicativo de “2” que
1
1 1 1 es “ ” a ambos lados de la desigualdad.
c) − 4.  ≤ 2 x.  ≤ 14.  2
2 2 2 d) Se encuentra el intervalo solución y se procede a
d) − 2 ≤ x ≤ 7 graficar.

S= [ − 2,7] ///////////////////////////////////////////
-2 0 7
Ejemplo Nº 3: 3( X − 2 ) ≤ 5 X + 8
Solución:
3( X − 2 ) ≤ 5 X + 8 ///////////////////////////////////////////
-7 +∞
3X − 6 ≤ 5X + 8
3X − 6 + 6 ≤ 5X + 8 + 6 S= [ − 7,+∞ )
3 X ≤ 5 X + 14
3 X − 5 X ≤ 5 X − 5 X + 14
− 2 X ≤ 14
 1  1
− 2 X . −  ≤ 14. −  X ≥ −7
 2  2

Sistemas de inecuaciones:
Son conjuntos de inecuaciones lineales con una incógnita que deben verificarse
simultáneamente; es decir, sea cual sea el número de conjuntos soluciones, todos deben
intersectarse para determinar el conjunto solución de dicho sistema, en conclusión son
todos los valores de x que satisfacen todas las inecuaciones. A continuación podemos
observar algunos ejemplos:
49
3 x − 6 < 6
Ejemplo Nº 4: 
− x + 3 ≤ 0
Solución:
Se debe resolver de manera independiente cada una de las inecuaciones del sistema
planteado.
Primera Inecuación S1 Segunda Inecuación S2

3x − 6 < 6 − x+3≤ 0
3x − 6 + 6 < 6 + 6 − x +3−3 ≤ 0−3
3 x < 12 − x ≤ −3
1 1 ( − 1) − x ≤ ( − 1) − 3
3x  < 12. 
 3  3 x≥3
x<4

S1
S2
////////////
3 4
St = S1 ∩ S2 = ( − ∞,4) ∩ [ 3,+∞ ) = [ 3,4)
El intervalo solución está dado por la intersección de las dos soluciones; es decir, si la
solución a la inecuación Nº 1 son todos los valores menores que 4 ( − ∞,4) y la solución
de la inecuación Nº 2 son todos los valores mayores o iguales a 3 [ 3,+∞ ) , entonces la
solución al sistema de las dos inecuaciones son todos los valores mayores o iguales a 3
pero menores de 4.

2 x − 7 > 13
Ejemplo Nº 5: 
3 x + 2 ≤ 4
Solución: Se debe resolver de manera independiente cada una de las inecuaciones del
sistema planteado
Primera Inecuación S1 Segunda Inecuación S2
2 x − 7 > 13 3x + 2 ≤ 4
2 x − 7 + 7 > 13 + 7 3x + 2 − 2 ≤ 4 − 2
2 x > 20 3x ≤ 2
1 1 1 1
2x.  > 20.  3 x. ≤ 4.
2 2 3 3
x > 10 4 4
x≤ S2 = (- ∞, ] ,
S1= (10,+ ∞) 3 3
S2 S1

50
4
10
3
Como la intersección es vacía, no hay solución para el sistema.
Ejercicios propuestos:
Determina el conjunto solución de las siguientes inecuaciones y realiza la representación
gráfica:
13.1) 3 x ≥ 13 13.2) 4 x + 5 > 2( x − 3)
x−3 3x − 1 4 x + 2
13.3) ≤2 13.4) <
2 2 3
Determina el conjunto solución de los siguientes sistemas de inecuaciones y represéntalo
gráficamente:
7 x − 2 < 3 x + 5 3 x + 1 < 2 x − 2
13.5)  13.6) 
2( x − 1) ≤ 3( x + 3) 2 x − 2 < x − 3
 x − 3 x +1  x − 2 < 3x + 5
 > 
13.7)  3 2 13.8)  2( x + 1)
 x < 1  3 − 5 ≥ 2
Expresa en forma de ecuación los siguientes problemas, resuélvelos y representa
gráficamente:
13.9) ¿Cuál es el entero positivo mayor que 80, cuyo tercio aumentado en 15 es mayor
que la mitad aumentado en 1?
13.10)La altura de un triángulo mide 10 metros. ¿Cuánto medirá la base de ese
triángulo, si su área no puede exceder de 50 metros?
13.11)La agencia de alquiler de automóviles A cobra Bs. 200.000 por día, más Bs. 1.000
por cada Km. La agencia B cobra Bs. 175.000 por día, más Bs. 1.750 por Km. ¿A
partir de qué kilometraje es más ventajoso el plan de cada agencia?
13.12)Las aspas de un ventilador dan una vuelta en 1/20 de segundo. ¿Cuál es el tiempo
necesario para que las aspas den más de 1000 vueltas?

51
LECTURA Nº 14: INECUACIONES EN LA RECTA

Tomado con fines instruccionales de:

Fundación Polar. Inecuaciones en la recta. [Artículo


en línea]. Disponible:
http://www.fpolar.org.ve/matemática. Extraído: enero
12, 2007

Un fabricante de tornillos recibe un pedido de un cliente, el cual estipula que los tornillos
deben tener una longitud de 7,62cm y son aceptables siempre y cuando el error no
exceda al 5%. El error ocurre tanto si el tornillo es más largo, como si es más corto que lo
deseado. Como el 5% de 7,62cm es 0,381cm, entonces los tornillos son aceptados por el
cliente cuando su longitud no es menor que 7,239cm. Asimismo, la longitud de los
tornillos no debe ser mayor que 8,001 cm.
La menor longitud aceptable: (7,62 - 0,381) cm = 7,239 cm.
La mayor longitud aceptable: (7,62 + 0,381) cm = 8,001 cm.
Si representamos mediante la variable L la longitud (en centímetros) de los tornillos, lo
anterior se expresa simbólicamente así:
L1 7,239 cm L 2 8,001 cm
Gráficamente estas inecuaciones se representan de la siguiente manera:
Rango de variación permitido ± 5%
-0,381 +0,381

Tamaño exigido 7,62 cm

El margen de error aceptable en la fabricación del tornillo, es un intervalo cerrado que va


desde:
La medida exigida menos el 5% (7,62 - 0,381)cm = 7,239 cm. y
La medida exigida más el 5% (7,62 + 0,381) cm = 8,001 cm.

////////////
0 7,2390 8,001

52
Esta expresión representa la combinación de las dos inecuaciones anteriores y determina
el intervalo cerrado (7,239 ; 8,001); (7,239 , ∞ ) ; (-∞ , 8,001).
Es intervalo cerrado porque hasta ese valor se acepta la medida.
Las inecuaciones cuadráticas
Ya hemos realizado estudios de las inecuaciones lineales, cómo se resuelven y algunas
aplicaciones. Ahora nos tocará estudiar las inecuaciones cuadráticas no sin antes
recordarles que una inecuación se caracteriza por tener como marco de referencia las
relaciones de orden o desigualdades, junto con sus propiedades, además de poseer
como posible solución, un conjunto infinito de números reales o intervalos.
Las inecuaciones cuadráticas, al igual que las lineales o de primer grado, son
expresiones polinómicas cuya solución va a depender de los valores numéricos o raíces
que anulen al polinomio que las define. Se diferencian en que el polinomio de la
inecuación lineal es de primer grado y el de la inecuación cuadrática es de segundo
grado.
Veamos algunos ejemplos de inecuaciones cuadráticas:
3 x 2 ≥ 27 , x 2 − 2 > 23 , ( x + 3)( x − 1) < 0 , 5 x 2 − 2 x + 11 ≤ 0
Resolver una inecuación cuadrática, es un proceso que se puede hacer por muchos
métodos, desde un simple despeje, pasando por sencillos procedimientos de
factorización, hasta por aplicar la fórmula que la resuelve. Vamos a explicar algunos de
ellos de manera detallada:
Cuando se trate de un simple despeje:
Se tiene la inecuación 3x 2 + 2 ≤ 29 ,
Entonces;
Lo primero es hacer de la inecuación una ecuación; es decir,
⇒ 3x + 2 − 2 = 29 − 2
2
cambiamos el signo de la desigualdad por una igualdad, para
encontrar las raíces o parámetros de los intervalos.

3x 2 27 Luego, despejamos la variable “ x ” siguiendo los


⇒ 3 x 2 = 27 ⇒ = procedimientos de acuerdo a propiedades de la adición, la
3 3
multiplicación y la potenciación

Ya encontradas las raíces o parámetros de los intervalos,


⇒ x2 = 9 ⇒ x2 = 9 procedemos a hacer un gráfico de referencia con una parábola
y una recta numérica donde se registren los valores de las
raíces.

53
⇒ x = ±3 , esto es; x = +3 o x = −3

Ahora bien, si analizas el gráfico


podrás observar que hemos tomado
como solución el intervalo de la
recta que está entre los parámetros
“-3” y “+3”, pues la inecuación mantuvo constantemente el signo de la desigualdad
“menor o igual que” ( ≤ ), lo cual indica que los valores que satisfacen la inecuación se
encuentran por debajo de la recta internos a-∞la parábola.
-3 +3 +∞

Luego; la solución se expresa de la siguiente manera:

Sol = [ − 3,+3] La solución es un intervalo cerrado en ambos extremos, porque


así lo indica el signo de la desigualdad “ ≤ ”

-Cuando se trate de una factorización:

Se tiene la inecuación x2 + 6x + 5 > 0

Entonces; x 2 + 6 x + 5 = 0 Lo primero es hacer de la inecuación una ecuación;


es decir, cambiamos el signo de la desigualdad por
Es una ecuación de segundo grado, una igualdad, para encontrar las raíces o
parámetros de los intervalos.

Se tiene el polinomio Para resolver una ecuación de segundo grado es posible


factorizar el trinomio, para ello tenemos que recordar los
x 2 +6 x +5 procedimientos aplicados en la unidad Nº 1, de la selección
de lecturas en el contenido de factorización de trinomios.
x 5
Observemos resumidamente cómo se hace:
x 1

x + 5x = 6 x
Las expresiones internas a los rectángulos redondeados se
( x + 5) toman formando una adición con sus respectivos signos. Luego,
estos binomios se multiplican para establecer nuevamente la
( x + 1) igualdad, de la siguiente manera:

Luego,
Tenemos una multiplicación de dos factores desconocidos
( x + 5) ⋅ ( x + 1) = 0 igualados a cero (0). Para que esta igualdad resulte cero (0)
puede pasar que uno de los factores sea cero (0) o ambos lo
sean. Es decir; en a ⋅b = 0 , a = 0 o b = 0 , de acuerdo a
esto se tiene que: 54
( x + 5) = 0 ∨ ( x + 1) = 0
Luego, despejando la variable en cada igualdad, se obtiene:
x = −5 ∨ x = −1 De esta manera hemos obtenido los valores de las raíces o
parámetros de los intervalos. Ahora con estos valores hacemos
una representación mediante la parábola y la recta numérica.

En el gráfico se puede observar que se ha


tomado como solución la parte superior
de la recta; es decir, los intervalos
externos a los parámetros “-5” y “-1”,
pues, así lo indica el signo de la
Luego, la solución se expresa de la desigualdad “mayor que” ( > ) que se ha
siguiente manera: mantenido en la inecuación hasta el final.
-∞ -5 -1 +∞ Lo que indica que la multiplicación de
Sol = ( − ∞,−5) ∪ ( − 1,+∞ ) ( ) (
los dos factores: x + 5 y x + 1 es )
siempre positiva.

Te proponemos resolver algunos ejercicios, para que pongas en práctica los


conocimientos adquiridos hasta ahora sobre las inecuaciones cuadráticas:
14.1- x 2 − x − 20 ≥ 0 14.2- t 2 + 7 ≥ 8t
14.3- 9 x 2 − 6 x + 1 ≤ 0 14.4- d 2 + 21 > 10d
14.5- x 2 + x + 1 < 0 14.6- m 2 + 5m ≤ 0

55
UNIDAD 3
GEOMETRÍA Y TRIGONOMETRÍA
LECTURA Nº 15: ALGUNOS SISTEMAS DE MEDIDA

Tomado con fines instruccionales de:


Martínez, M.(1998). Mi primera enciclopedia científica
Matemática. Editorial del Valle de México, S.A. (p.40).
México:

A lo largo de la historia, se han establecido diversas referencias de medida que han


permitido estandarizar representaciones de longitud, volumen, tiempo, velocidad, en fin
múltiples formas de medir. En la historia se dice que los romanos utilizaban sus pies para
medir distancias; en las mediciones más pequeñas utilizaban el ancho del dedo pulgar el
cual ellos llamaban “uncía”.

Las longitudes muy largas las medían con pasos. Un paso comprendía dos etapas, una
con el pie derecho y la otra con el pie izquierdo. En las distancias de mayor prolongación
utilizaban las “millas”, una milla era equivalente a 1000 pasos, de allí la palabra milla
que proviene del latín “mille” que significa “mil”. Las millas, yardas, pies y pulgadas son
medidas del sistema imperial de medición; es curioso mencionar que el rey Enrique I
(1068-1135) creó una medida que sirviera a todos, era la distancia desde su nariz hasta
su pulgar y lo llamó “yarda”.
En nuestros días, una gran cantidad de países utilizan una medida estándar llamada
metro, que es mucho más extenso que una yarda. La unidad metro, tanto en España
como en Venezuela, miden lo mismo. El Sistema Internacional de Medidas (S.I.M.) utiliza
el kilómetro para distancias largas, el centímetro y el milímetro para distancias mucho
más pequeñas. A continuación, podemos observar algunas referencias antiguas y
modernas con respecto a las unidades de medidas:
Romano Métrico Imperial
1 milla = 1000 pasos 1 kilómetro = 1000 metros 1 milla = 1760 yardas
1 paso = 5 pies 1 metro = 100 centímetros 1 yarda = 3 pies
1 pie = 12 uncias 1 centímetro = 10 milímetros 1 pie = 12 pulgadas

56
LECTURA Nº 16: EL SISTEMA MÉTRICO DECIMAL

Tomado con fines instruccionales de:

Santamaría, J (2007) El Sistema Métrico Decimal.


Artículo no publicado. (Pp. 5). Tinaquillo, Estado
Cojedes.

En 1790, la Academia Francesa de Ciencias fue la que se encargó, de acuerdo a


lineamientos de la Asamblea Nacional Francesa y la proposición de los políticos
Talleyrand y Prieur, de establecer un sistema unificado de medidas de aplicación sencilla,
que culminó el 19 de marzo de 1791 con la definición de Sistema Métrico Decimal a partir
de las propuestas de dos comisiones. La unidad de longitud, el metro, se definió igual a la
diezmillonésima parte del cuadrante del meridiano terrestre. Los franceses, Pelambre y
Méchain fueron los encargados de medir el arco del meridiano terrestre que pasa por
Paris, comprendido entre Dunkerque y el castillo de Monjuich en Barcelona.
A partir de la unidad fundamental, el metro, se definieron todas las otras unidades: las de
superficie, las de volumen, las de peso y las de capacidad. Por ejemplo, el gramo se
definió, para la época, como el peso de la masa de un centímetro cúbico de agua
destilada, pesada en el vacío, a la temperatura de 4º C.
El Sistema Métrico Decimal es un Sistema, porque comprende un conjunto de medidas
relacionadas entre sí; es métrico porque su unidad fundamental es el metro; y es decimal,
porque sus medidas aumentan y disminuyen en potencia de 10.
Tanto en las medidas de longitud como en las demás, se utilizan múltiplos y submúltiplos
a partir de la unidad.
Para los submúltiplos se asignaron prefijos latinos: deci para diez; centi para cien; mili
para mil y así sucesivamente. Mientras que para los múltiplos se estableció el uso de
prefijos griegos: deca para diez; hecto para cien; kilo para mil, etc.
Para transformar, medidas de longitud de una magnitud a otra vamos a utilizar la
siguiente estrategia:
Km
Hm Se divide entre 10 por
Dam cada escalón que subes

Metro
dm
Se multiplica por 10 por cm
cada escalón que bajes
mm

Veamos algunos ejemplos:

57
Ejemplo 1:
Transformar:
35,328 Km a m
Si verificamos la escalera anterior, para pasar de Kilómetros a metros, tenemos que bajar
tres escalones. Entonces, según el procedimiento debemos multiplicar por 10, por 10 y
por 10. Es decir:
( 35,328).10.10.10 = ( 35,328).(10 3 )
Recuerda, cuando se multiplica una cantidad por una potencia de
base 10 se corre la coma hacia la derecha tantos espacios como lo
indique el exponente de la potencia.

Por lo tanto, ( 35,328).10 = 35328,0


3

Observa que la cantidad tiene tres decimales y se está multiplicando por 10 3 , la coma se
corrió a la derecha tres, espacios, esto hace que la cantidad quede sin decimales:
( 35,328 ).10 3 = 35328
∴ 35,328 Km a m es 35328 metros.
Ejemplo 2:
Transformar:
21307 mm a Dm.
Si verificamos la escalera anterior, para pasar de milímetros a decametros, tenemos que
subir cuatro escalones. Entonces, según el procedimiento debemos dividir por 10, por 10,
por 10 y por 10. Es decir:
21307 21.307
=
10.10.10.10 10 4

Recuerda que cuando se divide una cantidad por una potencia


de base 10 se corre la coma hacia la izquierda tantos espacios
como lo indique el exponente de la potencia.
Si la cantidad es un número entero la coma se omite, pero podemos agregarle la coma
para indicar que tiene cero (0) decimales, así: 21307,0
Luego:
21307,0
= 2,13070
10 4

Observa que la coma se corrió hacia la izquierda, cuatro espacios de acuerdo al


exponente de la potencia de base 10.
Por lo tanto:
58
21307 mm a Dam es 2,1307 Decametros.
Te proponemos algunos ejercicios para que practiques este procedimiento de conversión
de medidas:
15.1. 3584,1 dm a Dam 15.2. 1,435 Km a cm
15.3. 0,000153 Hm a mm 15.4. 58973,003 cm a Hm
15.5. 3 dm a m 15.6.1 m a Dam
Los múltiplos y submúltiplos en el sistema métrico decimal, se justifican por lo siguiente:
Imagínate a un sastre diseñando la manga de una camisa, es de suponer que necesitará
cierta cantidad de tela, y expresarla en centímetros bastará para satisfacer los
requerimientos.
Mientras que es diferente en el caso de un ciclista profesional, sus actividades o
recorridos son en grandes distancias, estas cantidades bastaría expresarlas en
Kilómetros y no en centímetros; pues no es que no se pueda, pero no sería lo adecuado.
Las medidas de superficie son las mismas utilizadas en las longitudes, a diferencia de
que aquí se expresan en unidades cuadradas. Por ejemplo, para expresar el área de un
terreno se puede hacer en metros cuadrados (m2) o kilómetros cuadrados (Km2).
Recuerda que las superficies se representan en dos dimensiones.
Para realizar conversiones de medidas de superficie se puede aplicar el procedimiento de
la escalera; pero debes tener cuidado, pues las medidas aumentan o disminuyen en
potencias de 100, es decir 102.
Ocurre lo mismo con las medidas de capacidad, cuyas medidas nos dan a conocer el
volumen de un cuerpo. Se sabe que un cuerpo tiene tres dimensiones, por tal motivo, al
hablar del volumen de una caja, de un tanque, entre otros; se puede representar en
centímetros cúbicos, metros cúbicos, etc.
Las conversiones que se realizan en medidas de capacidad aumentan o disminuyen en
potencias de 1000, es decir, 103.
Multiplicas Multiplicas por
Km2 por 102 por Km3 103 por cada
Hm2 cada escalón Hm3 escalón que bajes
que bajes
Dam2 Dam3
m2 m3
dm2 dm3
Divides por 102 cm2 cm3
Divides por 103
por cada escalón
mm2 por cada escalón mm3
que subas que subas

Escalera para transformar medidas de Escalera para transformar medidas de capacidad


superficie.

59
Revisemos algunos ejemplos sobre conversiones de medidas en superficie y de
capacidad.
Ejemplo 1.
Transformar 12 m2 a cm2
Según la escalera, para pasar de metros a centímetros se tiene que bajar dos escalones,
entonces se debe multiplicar la cantidad dada por 100, y por 100, es decir:
(12).100.100 = (12).10 2.10 2
(12).10 2.10 2 = (12).10 4
(12).10 4 = 120000
Por lo tanto:
12 m2 a cm2 = 120000 cm2
Ejemplo 2:
Transformar: 3,5 cm3 a m3
De acuerdo a la escalera, para pasar de centímetros a metros hay que subir dos
escalones, por lo tanto se debe dividir por 1000, y por 1000, esto es;
3,5 3,5 3,5
= 3 3
= 6
1000.1000 10 .10 10

Según el procedimiento, cuando se divide por una potencia


de base 10 se corre la coma hacia izquierda tantos espacios
lo indique el exponente de la potencia.

Entonces;
3,5
= 0,0000035
10 6

En conclusión:
3,5 cm3 a m3 = 0,0000035 m3
Resuelve los siguientes ejercicios para que adquieras un mayor dominio de tus
habilidades:
Realiza las siguientes conversiones de unidades y resuelve los problemas
planteados:
15.7.5,823 Dam3 a cm3 15.8.0,0045 m3 a Km3
15.9.8 dm2 a mm2 1 2
15.10. m a Hm2
5

60
1 1
15.11. Km3 a Dam3 15.12. Hm2 a cm2
100 1000
3
15.13. mm3 a Km3
8
15.14.Un maratonista, para su entrenamiento, realiza durante cinco días los siguientes
recorridos: el primer día recorre 950 Dam, el segundo día 122 Hm; en el tercer día
14 Km, en el cuarto 15420 m, y para el último día recorre 1.800.000 cm. ¿Cuántos
kilómetros recorre en los cinco días?
15.15.Calcula la diferencia que existe entre un recipiente, cuya capacidad es de 54 m3
y otro de 44.100.000 cm3
Se sabe que la unidad de volumen en el Sistema Internacional de Medidas es el metro
cúbico (m3), pero existe otra unidad de medida para representar las capacidades de los
cuerpos como lo es el litro (l). Se relaciona con la otra unidad, ya que 1 decímetro cúbico
(1 dm3) es equivalente a 1 litro de agua pura a temperatura de 4º C. Litro, centilitro,
mililitro, son medidas de capacidad que tienen sus equivalentes en volumen; por ejemplo:

1 m3 = 1000 dm3 = 1000 Litros


1 dm3 = 1000 cm3 = 1 Litro
100 cm3 = 100 militros
1cm3 = mililitro
Si nos vamos a situaciones de la vida cotidiana; en varios productos es frecuente
expresar sus cantidades en cm3 (abreviado cc) o en mililitros (ml). También es usual en
muchos productos importados: perfumes, cosméticos, medicinas, etc., expresar las
cantidades del producto (capacidades netas de los recipientes que los contienen) en una
unidad inglesa expresada como fl oz (onza de flido). Por ejemplo: 16,9 fl oz (500 ml); 4,2
fl oz (125 ml), tal cual como se lee en las etiquetas de esos productos. ¿Cuántos ml
equivalen a 1 fl oz?
Realiza las siguientes conversiones:
15.16.3240 ml a m3 15.17.53 dm3 a ml

61
LECTURA N° 17: FIGURAS POLIGONALES
Tomado con fines instruccionales de:
Suárez, E. y Cepeda, D. (2003). Matemáticas de
Educación Básica. Editorial Antillana, S.A. (p.149)
Caracas, Venezuela:

Un polígono es la parte del plano limitada por una línea poligonal cerrada.
Fíjate en el siguiente polígono:
Plano

• Los segmentos AB, BC, CD, DE, EF, FG y GA se denominan lados.


• El vértice de un polígono, es el punto de intersección de dos segmentos o lados. Dos
vértices son consecutivos si son extremos de un mismo lado. Los vértices se denotan
así: vértice A, vértice B, etc. Este polígono tiene 7 vértices.
• El ángulo interno de un polígono, es la abertura formada por dos lados en un vértice.
Los ángulos denotan así: ∠BEC , ∠ABC , ∠FEB , ∠FED , ∠EBC , ∠EBA , etc. Hay
muchos ángulos en este polígono.
• La diagonal de un polígono, es el segmento de recta que une dos vértices que no
pertenecen a un mismo lado. Tenemos la diagonal BE, y podemos trazar en este
mismo polígono, diagonales entre los vértices: A y C, A y D, A y E, A y F, B y D, B y F,
etc. Se pueden trazar varias diagonales.
• El perímetro de un polígono se calcula sumando las medidas de las longitudes de
cada lado. El perímetro de este polígono es igual a:
P = AB + BC + CD + DE + EF + FG + GA .
Se habla de polígono convexo y cóncavo. Un polígono es convexo, si cada uno de sus
ángulos interiores es menor de 180º. Es cóncavo si uno de sus ángulos es mayor de
180º.

Polígono Cóncavo Polígono Convexo

62
Clasificación de los polígonos
Los polígonos se clasifican según sus lados en:
Número de lados Nombre del polígono
3 Triángulos
4 Cuadriláteros
5 Pentágonos
6 Hexágonos
7 Heptágonos
8 Octágonos
9 Eneágonos
10 Decágonos
11 Undecágonos
12 Dodecágonos

Un polígono es regular, cuando todos sus lados miden igual y todos sus ángulos también
son iguales.
La apotema de un polígono regular, es el segmento de recta que une centro del
polígono con el punto medio de uno de sus lados.

LECTURA N° 18: LOS TRIÁNGULOS, LOS


CUADRILÁTEROS Y SUS RELACIONES MÉTRICAS
Tomado con fines instruccionales de:
Fundación Polar. Matemática para todos. [Artículo en
línea]. Disponible: http://www.fpolar.org.ve/matemática.
Extraído: enero 12, 2007

LOS TRIÁNGULOS
El triángulo tiene una característica especial, es estable; por ello es vital en la industria,
en efecto, si a una estructura en forma de triángulo se le aplica una fuerza en uno de sus
vértices, la forma del triángulo permanece. Observa las estructuras de una torre utilizada
en la extracción de petróleo, o las torres que sostienen algunas antenas parabólicas, y
también en muchos edificios.
El triángulo es un polígono de tres lados. El triángulo ABC se
refiere al triángulo determinado por los puntos A, B y C. En este
caso sus lados son los segmentos AB, BC y AC. Los ángulos
del triángulo son los ángulos de vértices A, B y C, es decir,
∠ CAB, ∠ ABC y ∠ BCA.

63
El símbolo representa la palabra triángulo. Así ABC significa el triángulo ABC.
Clasificación de los triángulos
Según sus ángulos:

Acutángulo: Tiene tres Obtusángulo: Tiene un Rectángulo: Tiene un


ángulos agudos (miden ángulo obtuso (miden ángulo recto (mide 90º)
menos de 90º) más de 90º)

Según sus lados:

Escaleno: Todos sus


Equilátero: Tiene tres Isósceles: Tiene dos
lados miden distinto.
lados miden igual lados que miden igual

Otros elementos de los triángulos

Alturas: Segmento Bisectrices: Semirrecta Medianas: Segmento Mediatrices: Recta


desde cada vértice que divide cada ángulo desde cada vértice al perpendicular a cada
perpendicular al lado en dos ángulos iguales punto medio del lado lado en su punto medio
opuesto opuesto

Ortocentro: Punto de Incentro: Punto de Baricentro o Centro de Circuncentro: Punto de


intersección de las intersección de las gravedad: Punto de intersección de las
alturas bisectrices y centro del intersección de las mediatrices y centro del
círculo inscrito en el medianas círculo circunscrito al
triángulo triángulo

64
LOS CUADRILÁTEROS:
Los cuadriláteros son polígonos de cuatro lados, se caracterizan por tener, cuatro
lados; cuatro vértices, cuatro ángulos interiores, cuatro ángulos exteriores y dos
diagonales. Observen las figuras:
Lado A
Vértice A B Diagonales
C
Ángulo
exterior B D
D α
C
Ángulo
interior
Cuadrilátero
Cuadrilátero Cóncavo

En cuadrilátero convexo se muestra que:


- Los lados son los segmentos: AB, BC, CD, DA.
- Los vértices, son cada encuentro de los lados: A, B, C y D.
- Los ángulos internos, son cada abertura entre dos lados, son: * ∠ DAB, ∠ ABC, ∠
BCD, ∠ CDA. (Observa las tres letras, la que esta en el medio es de donde surge el
ángulo)
* El signo “ ∠ ” se lee ángulo

- Las diagonales, son cada segmento que une dos vértices opuestos, son: AC, BD.
- La letra griega “ α ” Alfa denota un ángulo exterior.
A un cuadrilátero se le puede calcular el perímetro y su área.
- El perímetro es la suma de las longitudes de sus cuatro lados:
Perímetro del cuadrilátero ABCD = AB + BC + CD + DA.
Sabemos que un triángulo tiene tres ángulos y la suma de las medidas de esos ángulos
es de 180º. Un cuadrilátero se puede dividir en dos triángulos trazando una de sus
diagonales, por tal motivo los cuatro ángulos del cuadrilátero al sumarse se obtiene 360º
A α B
La suma de los cuatro ángulos del cuadrilátero:
ABCD: β ∠DAB + ∠ABC + ∠BCD + ∠CDA = 360º
δ σ O también: Letras griegas:
α + β + δ + σ = 360º α se lee Alfa
β se lee Beta
D C
δ se lee Delta
σ se lee Tita

65
Clasificación de los Cuadriláteros:
CUADRILÁTEROS FIGURA Y DIAGONALES

Rectángulos A B

D C
A B
Cuadrado
D C
Paralelogramos
A B
Rombo C
D
A B

C
Romboide D
A B

Trapecio D C
Rectangular

A
Trapecios Trapecio B

Isósceles
A B C
D

Trapecio
D C
Escaleno
A
Trapezoides B

D
C

Ejercicios propuestos: calcula el perímetro en cada figura.

18.1.Cuadrado de lado 2/3m. 18.2.Un rectángulo formado con las unión


de dos cuadrados de lado 8 m.
Triángulo isosceles
18.3. 18.4.Un rombo formado por la unión de
7m
10 m dos triángulos equiláteros de lado x/2
6m

22 m

LECTURA N° 19: LA CIRCUNFERENCIA Y SUS


ELEMENTOS
66
Tomado con fines instruccionales de:

Santamaría, J (2007). La Circunferencia y sus


Elementos. [Artículo no publicado]. (Pp. 2).
Tinaquillo, Estado Cojedes.

La circunferencia, es el conjunto infinito de puntos que están a una misma distancia de un


punto fijo llamado centro. La distancia del centro a cualquier punto de la circunferencia
se le llama radio.
Elementos de una circunferencia:
La distancia del centro al punto R o segmento
OR es un radio de la circunferencia.
La distancia del punto P al punto Q o segmento
PQ es un diámetro de la circunferencia. Un
diámetro equivale a dos veces el radio.
La distancia del punto A al punto B o segmento
AB es una cuerda de la circunferencia.
La recta “s” que toca dos puntos, el punto M y el
punto N, de la circunferencia es una recta
secante a la circunferencia.

La recta “t” que toca un solo punto, el punto P, de la circunferencia es una recta tangente
a la circunferencia.
El conjunto de puntos que pertenecen a la circunferencia y están entre dos puntos de
ella, entre el punto A y el punto B, por ejemplo, se le llama arco de la circunferencia. El
arco de extremos A y B se denota arco
Los puntos R, O y P describen un ángulo central a la circunferencia, y se denota ∠ROP
.
El conjunto infinito de puntos que forman la circunferencia y los interiores
a ella conforman una superficie llamada círculo.
La región comprendida entre los puntos A, B y O, o mejor dicho, todos los
puntos interiores al ángulo ∠AOB representa un sector circular de dicho
círculo.
A una circunferencia es imposible calcularle el área, pues sólo
representa una línea cerrada que limita al círculo, a la circunferencia se
le puede calcular la longitud y al círculo se le calcula el área. La fórmula
para hallar la longitud de una circunferencia es: L = π ⋅ 2r , siendo

67
π ≈ 3,14 y r = radio de la circunferencia y para determinar la longitud un ángulo central se
utiliza:
π ⋅ r ⋅ nº
L= .
180º

Mientras que la fórmula para hallar el área de un círculo es: A = π ⋅ r 2 .


Y para calcular el área de un sector circular se usa:

π ⋅ r 2 ⋅ nº
A= , donde nº representa la amplitud del ángulo.
360º

El ángulo central de una circunferencia es aquel que está formado por dos de sus radios.
Cada ángulo central determina una cuerda y un arco, y a la vez cada cuerda determina
un arco y un ángulo central, y un arco determina un ángulo y una cuerda. Observen la
figura, allí se describe en el ángulo central ∠DOE , el arco y la cuerda DE. La medida
de amplitud de un arco de una circunferencia se representa en grados (º), y la de un
ángulo central de la misma manera; ya habíamos dicho que un ángulo central determina
un arco y viceversa, esto indica que las medidas en grados para ambos son iguales. Es
decir, si un arco mide 60º, su ángulo central mide 60º.
Ejercicios :
19.1.¿Cuál es la longitud de una circunferencia que tiene de radio 5 Km?
19.2.¿Qué longitud tiene un arco cuya amplitud del ángulo central es de 30º?
19.3.Determina el área de un sector circular, si su ángulo central es de 22º.
19.4.Calcula el área de un círculo cuyo diámetro es de 25 metros.
19.5.En una semicircunferencia el radio es de 3/2 cm ¿cuál es su longitud?
1
19.6.La longitud del un arco de una circunferencia es de π , calcula la medida de su
3
ángulo central.
19.7.Hallar el diámetro de un círculo, sabiendo que su área es igual a 100π.

68
LECTURA N° 20: CUERPOS GEOMÉTRICOS Y SUS
ELEMENTOS

Figura 7
Tomado con fines instruccionales de:
Santamaría, J (2007). Los Cuerpos Geométricos y sus
Elementos. Artículo no publicado. (Pp. 3). Tranquillo,
Estado Cojedes.

En la geometría plana, se estudian aquellas figuras y formas geométricas que tienen una
o dos dimensiones; y sólo se pueden representar en una superficie plana, como la
circunferencia, el círculo, el triángulo, los cuadriláteros y demás polígonos.
La geometría del espacio se encarga de estudiar aquellas formas, cuerpos y objetos que
tienen tres dimensiones. Estas formas se encuentran en el mundo real, sea de manera
artificial, construidas por el hombre, como por ejemplo: edificaciones, herramientas,
envases, entre otros y la que pertenecen a la naturaleza, como: árboles, montañas, roca,
planetas, animales, seres humanos.
CARACTERÍSTICAS DE ALGUNOS CUERPOS GEOMÉTRICOS
Cuerpos Redondos:
La esfera: es un cuerpo cuya superficie es curva, carece de vértices y su volumen se
calcula mediante la fórmula:
4
V = π .r 3
3

Si hacemos un corte a una esfera con un plano obtenemos una circunferencia. Observa
la figura:
Plano

Circunferencia

C r Circunferencia
máxima
P

La distancia de C a P es el
Radio

Si la esfera es sólida como una bola, al realizar el corte obtendríamos el círculo.


El Cilindro: es un cuerpo mixto; es decir, tiene superficie plana y superficie curva. El
cilindro consta de una base circular y de una determinada altura. Su volumen se halla
mediante la fórmula. V = Área de la base x Altura.
69
Si hacemos un corte al

Eje
cilindro con un plano
paralelo a la base, se
obtiene un círculo. Si el
corte se hace
perpendicular a la base, se
Base obtendría un rectángulo.
Base

El Cono: es un cuerpo de base plana y de superficie lateral curva. A diferencia del


cilindro, el cono sólo tiene una base y tiene un vértice. El volumen de un cono se calcula
mediante la fórmula:

π .r 2 .( Altura )
V = Vértice
3
Si hacemos un corte con un plano
paralelo a la base del cono se
obtiene un círculo

Eje Base

Poliedros: Muchas edificaciones construidas por los humanos y algunos cuerpos de la


naturaleza tienen forma de poliedros. Los poliedros son cuerpos limitados por un número
finito de superficies planas. Estas superficies planas son polígonos que reciben el nombre
de caras del poliedro. La intersección de dos caras forman una arista y el punto de
intersección de tres o más caras es un vértice.

Entre los poliedros se encuentran: Las pirámides y los prismas.


Las pirámides: Son poliedros cuyas caras laterales tienen forma de triángulo; el número
de triángulos o caras laterales de una pirámide, depende del número de lados de la base.
Éstas pueden ser de base triangular, cuadrada, pentagonal, etc. Los triángulos que

70
conforman las caras de la pirámide convergen en un punto, es decir, tienen un punto en
común; este punto recibe el nombre de vértice de la pirámide.
Vértice

Arista
Cara

Base cuadrada
Pirámide Hexagonal
Pirámide cuadrada

Los prismas: son cuerpos geométricos tridimensionales, la característica más


sobresaliente es que dos de sus caras son paralelas (caras opuestas) y congruentes,
llamadas bases del prisma. Cada prisma recibe su nombre de acuerdo a la forma de sus
bases.
Los prismas, cuyas caras laterales son rectángulos, son llamados prismas rectos; de otra
forma son llamados prismas oblicuos.
Los prismas rectangulares o “cajas” también son llamados paralelepípedos.
Veamos algunos prismas:

Base cuadrada

Vértice

Cara lateral Bases


Triangulares

Arista
Prisma Triangular

Algunas cosas curiosas de la naturaleza guardan relación con estas formas geométricas,
por ejemplo: ¿Has llegado a ver de cerca un panal de abejas? Si lo observas
detalladamente parece un piso cubierto de mosaicos hexagonales. Pero su forma
tridimensional es la de prismas rectos hexagonales. Entre el triangulo equilátero, el
cuadrado y hexágono regular, este último tiene el menor perímetro para un área
establecida. Esto significa, que en los panales de abejas en forma de prisma hexagonal
se usa menos cera para su construcción.

71
LECTURA Nº 21: EL NÚMERO PI ( π ) Y EL CÁLCULO DE
ÁREAS

Tomado con fines instruccionales de:

Fundación Polar. El número pí () y el cálculo de áreas. [Artículo


en línea]. Disponible: http://www.fpolar.org.ve/matemática.
Extraído: enero 7, 2007

El número π (Pi), tiene un origen un poco extenso y muy apasionante; en la antigua


Grecia, su aparición se relacionó con el resultado de dividir la longitud de una
circunferencia entre la longitud de su diámetro, por lo que se denota con letra griega π,
inicial de la palabra “περιμετρο” que significa perímetro. Leonard Euler (1707-1783),
matemático suizo, fue quien hizo famosa la notación de π, a pesar de haberla
implementado en sus estudios William Jones muchos años antes.
La aproximación al número π se remonta a las civilizaciones más antiguas, ejemplo de
ello fueron los babilonios y egipcios, que aún cuando desconocían su nombre y
simbología, le atribuyeron el valor “3” obtenido con la aproximación de la longitud de una
circunferencia mediante “6r” que es el perímetro del hexágono regular inscrito.

Es decir, de la relación r
Lado del hexágono = radio
6r = 2πr, se obtiene que π = 3 r de la circunferencia
r=r

Hay un pasaje de la Biblia donde también se puede deducir ese valor “3”:
“…Él, hizo también un vaso de metal fundido, la gran cuba, que
tenía diez codos de diámetro y era perfectamente redondo, y tenía
cinco codos de alto, en tanto que un cordón de treinta codos
medía la circunferencia”.
De aquí se cumple que: π = 30 codos/10 codos = 3.
Aún en nuestra era se hacen cálculos sobre π, llegando a representarlo con 109 cifras
decimales. Éste número es tomado en cuenta en muchas fórmulas matemáticas
relacionadas con medidas: longitud de una circunferencia, área de un círculo, área de un
óvalo, volumen de un cilindro, de un cono y de una esfera, área de la superficie de una
esfera, entre otros.
El primer matemático que hizo cálculos de π con muchas cifras, 707 cifras decimales, fue
el inglés William Shanks en 1873, cifras que adornan la cúpula del “Palacio del

72
Descubrimiento” en el Museo de Ciencias de Paris. Esta cúpula se encuentra en una
sala que tiene 10 metros de diámetro y π decámetros de perímetro.
El matemático e ingeniero venezolano Francisco José Duarte (1883-1972), nacido en
Maracaibo, también calculó el número π con muchas cifras. Duarte escribió, en 1956, una
monografía sobre los números irracionales π y ℮

Procedimientos para calcular áreas y volúmenes de figuras y cuerpos


geométricos:
En muchas labores de la vida cotidiana se deben hacer cálculos para determinar el área
de una determinada región; ya sea sobre un terreno que se va a cultivar, alguna
edificación que se va a construir; sobre un piso que se va a cubrir con alfombra o
cerámica; sobre una pared o un lienzo donde se va realizar una pintura, entre otros.
También es importante realizar cálculos de volumen en situaciones donde; se necesite
saber, cuántos litros de agua requiere una piscina, un tanque, una botella o cualquier otro
envase; la cantidad de cajas que ocupan una habitación o cava; entre otras actividades
de la vida diaria.
El área de una figura plana, es la medida de la región encerrada por líneas poligonales,
en otras palabras es la medida de la superficie.
Realicemos algunos cálculos de perímetro y área.
Ejemplo 1:
En el terreno de béisbol, las cuatro bases forman un cuadrilátero como se muestra en la
figura. Si entre cada base hay una distancia de 90 pies, es decir, 27 metros ¿Cuántos
metros recorre el bateador al dar un jonrón? 2da Base

Solución:
3ra Base
Sólo tenemos que calcular el perímetro
del cuadrilátero: Recuerda que para 1ra Base
calcular el perímetro de un polígono se
suman las longitudes de sus lados.

Home
Entonces:
P = 27m + 27m + 27m + 27m
Luego Perímetro = 4.(27m) = 108 metros.
Por lo tanto: El bateador recorre 108 metros al dar el jonrón.
¿Calcula el área que hay entre las cuatros bases?
Solución: Para calcular el área del cuadrilátero que es un cuadrado, sólo debemos
multiplicar la medida de un lado dos veces, así.
73
Área = (Lado)2
Área = (27m)2 = (27m).(27m) = 729 m2
27m

El área del cuadrilátero que está entre las cuatro bases


es de 729m2
27m

Ejemplo 2:
El terreno de una siembra de café tiene forma de un trapecio isósceles, como se muestra
en la figura, se necesita saber el perímetro y el área del terreno.

A B
AB = 3Km
Donde: BC = 5 Km

AD = BC
Altura
2 −
D C − 11 AF = Altura
F DC = Km
2
− 8
AF =
3
Solución:
Para calcular el perímetro del trapecio, aplicamos la fórmula: P = AB + BC + CD + DA
5 11 5
Entonces: Perímetro = 3Km + Km + Km + Km
2 2 2

21 27
Perímetro = 3Km + Km = Km
2 2

27
El perímetro del terreno es de Km
2

Luego, cálculo del área:


El área de un trapecio se calcula mediante la fórmula:

Área =
( Base mayor + base menor ) ⋅ Altura
2

Donde: base mayor = DC; base menor = AB


Altura = AF.
Sustituyendo; queda:

74
 11 5   16 
 Km + Km   Km 
Área = 2 2  ⋅  8 Km  Área =  2  ⋅  8 Km 
   
2 3  2 3 

8KM  8  8 
Área = ⋅  Km  Área = 4Km ⋅  Km 
2 3  3 

32 32
Área = Km 2 El área del terreno es de Km 2
3 3

Ejemplo 3:
Una constructora, ha dividido un terreno en cuatro partes iguales para la
edificación de cuatro casas. Si el terreno tiene forma de rombo y las medidas y divisiones
se especifican en la figura dada. Calcular el área de todo el terreno y el área que
corresponde a cada casa.

D Donde:

DE = 10 Dam
G E −
DF = 16 Dam

F EG = 12 Dam
Solución: Para calcular el área de un rombo
se aplica la fórmula:
Diagonal mayor ⋅ diagonal menor
Área =
2

Donde; diagonal mayor = DF y diagonal menor = EG


Sustituyendo; queda:

Área =
(16Dam ) ⋅ (12Dam ) =
192Dam 2
Área = 96Dam 2
2 2

Luego el área de todo el terreno es igual a 96 Dam2


Para calcular el área de una de las divisiones, podemos dividir el área total entre 4 o
tomamos una de las cuatro divisiones; que representan triángulos y le calculamos el área.
D
Para calcular el área del DEC, se necesita conocer la
base “CE” y la altura “CD”.
G E base . Altura
C Recuerda que Área =
2

F
75
Como las diagonales de un rombo se cortan en sus puntos
medios, entonces: La mitad de la diagonal EG es igual a CE.
EG 12Dam −
= = 6Dam
Esto es: 2 2 También: DF = CD
CE = 6Dam = base 2

DF 16Dam −
Esto es; = = 8Dam CD = 8Dam = Altura
2 2

Por lo tanto; Área =


( 6Dam ) ⋅ ( 8Dam ) =
48Dam 2
= 24Dam 2
2 2

Ejemplo 4:
Miguel es albañil y quiere construir en el patio de su casa un caney de base pentagonal.
Si del centro de la superficie de la base, al punto medio entre dos columnas, la distancia
es de 7/2 metros y entre cada columna hay una distancia de 3 metros; ¿Cuál es el área
del pentágono?
Solución: Los vértices A, B, C, D, E
A Son los puntos donde van las columnas. El segmento FH es un
B apotema.
E
F
Como AB = 3m y el pentágono es regular, entonces,
D H C
7
AB = BC = ED = DC = CB y FH = m
2

Luego, para calcular el área de un pentágono regular se aplica la fórmula.


( Perímetro del polígono ) ⋅ Apotema
Área =
2

Entonces; Perímetro = AB + BC + CD + DE + AE
Pero como todos los lados miden igual
Perímetro= 5.( AB ) = 5.(3m ) = 15m
Por lo tanto;

(15m ) ⋅  7 m 105 2
m
Área = 2 = 2 105 2 es el área del pentágono.
= m
2 2 4

76
Cálculos de algunos volúmenes en cuerpos geométricos.
Ejemplo 5:
Una tarde, el joven Julio caminaba con su padre por cierta avenida y observa,
detalladamente, las cosas a su alrededor; le dice: Papá, viste que algunos carros, en la
parte trasera, llevan escrito algunos símbolos como: 1.3L, 1.6L, 2.0L, 4.5L, etc. ¿qué
significan esos números?
El padre, como todo un experto, le contesta: Hijo, esas expresiones hacen referencia a la
cilindrada del automóvil, en otras palabras al volumen útil de los cilindros; cuanto mayor
es la expresión que allí se indica mayor es la cilindrada del vehículo. Por ejemplo, en un
carro de cuatro cilindros, si calculamos el volumen de cada cilindro, mediante la fórmula:
V = π ⋅ r 2 ⋅ h , siendo π ≈ 3,1416 ; h = altura = 7,548cm y r = radio = 4,1035cm .
Sustituyendo la fórmula, queda:
V = (3,1416) ⋅ (4,1035cm) 2 ⋅ (7,548cm) = 399,29cm 3 , ésta representa la capacidad para
cada cilindro. Si el carro es de 4 cilindros, entonces la cilindrada es de:
4V = 4(399,29cm 3 ) = 1597,16cm 3 .
Redondeando esta cantidad por exceso nos resulta, que: V = 1600cm 3 , esto es
equivalente a decir V = 1,6litros , y se anota de esta manera para simplificar la escritura.
Ejemplo 6:
Una piscina, tiene la forma de un prisma como el que se muestra en la figura ¿cuántos
litros de agua se necesitan para llenarla por completo?
Solución:
Observa que si la piscina fuese un
paralelepípedo el volumen sería:
V = ( Áreabase) ⋅ altura
Esto es, V = (10m ⋅ 4m) ⋅ (4m)
Luego; V = 160m 3
Pero a la piscina le hace falta un pedazo, para ser un paralelepípedo,
algo como esta forma; un prisma triangular, cuyo volumen es:
( Área base)
V= ⋅ altura . Sustituyendo, queda;
2

( 4m ⋅ 4m ) ⋅ 5 2 m (16m )5m = 80m


2 3
V = = = 20m 3
2 4 4

Luego, al volumen del paralelepípedo le restamos el volumen del prisma triangular y nos
dará el volumen de la piscina, así: Volumen de la piscina = 160m 3 − 20m 3 = 140m 3
Pero nos piden la capacidad de la piscina en litros, por lo que hay que transformar 140m 3
a litros ; para hacer esto, primero tenemos que trasformar 140m 3 a dm 3 . De acuerdo

77
a la escalera de conversión, se tiene que: 140m 3 a dm 3 = 140000dm 3 , si se sabe que
1dm 3 = 1litro , entonces; 140000dm 3 = 140000litros .
Por lo tanto la piscina necesita 140000litros de agua para llenarse por completo.
Te proponemos algunos ejercicios y problemas, debes ejercitar todo lo relacionado al
cálculo de áreas y volúmenes de figuras y cuerpos geométricos.
Ejercicios:
21.1.¿Cuál es el área de un triángulo cuya base mide 35 cm y su altura es 3/5 de la
base?
21.2.¿Cuánto mide la base menor de un trapecio que tiene como área 204 m 2, la base
mayor es de 32 m y su altura es de 12 m?
21.3.¿Cuál es la medida de uno de los lados de un polígono regular de 16 lados, de
apotema igual a 60 cm y área 16000 cm2?
21.4.¿Cuál es el área de un círculo cuyo perímetro es de 52 dm?

Según la figura que se te indiqua a continuación, realiza los cálculos respectivos:


21.5. Si el área de la figura es igual a 68 cm2
¿cuánto vale b? b
9 cm.

21.6. El triángulo ABC es isósceles; si AD = 11 cm.


y CD = 7/2 cm. ¿cuánto vale el área?
C D B

El trapecio de la figura se ha construido con tres triángulos rectángulos, donde uno de


ellos es isósceles.
21.7. Halla el área del trapecio de dos
maneras: usando la fórmula del área 13 cm. 13 cm.
del trapecio y hallando la suma de las 12 cm. 12 cm.
áreas de los tres triángulos
rectángulos. 7 cm. 7 cm.
Resuelve los siguientes problemas:
21.8.La habitación de Juana, mide 4 m de ancho, 5 m de largo y 5/2 m de alto. El área
de la puerta y la ventana es de 2 m 2. Ella desea colocar papel tapiz a las cuatro
paredes; si cada rollo de papel mide 50 cm de ancho por 5 m de largo ¿Cuántos
rollos de papel necesitaría Juana para cubrir las paredes?
21.9.En Tinaquillo hay una estación de radio que tiene una cobertura igual a un radio
de 72 Km. ¿Cuántos kilómetros cuadrados cubre la señal de la estación de radio?
78
21.10.Carlos tiene un terreno de forma cuadrada, cuyo lado mide 18 m. En cada
esquina del terreno hay un poste y un caballo atado por una cuerda de 9 m. ¿Qué
parte del terreno no puede se recorrida por los caballos?
21.11.Calcula el volumen de los siguientes cuerpos:

21.13.El volumen de un cilindro es 330п cm3. Calcula el radio de la base si la altura


mide 6 cm.
21.14.Determina la altura de un cono que tiene un volumen de 108п m3 y el área de la
base es igual a 36п m2.

LECTURA Nº 22: THALES Y LA PIRAMIDE DE KEOPS

Tomado con fines instruccionales de:


Fundación Polar. Thales y la pirámide de Keops.
[Artículo en línea]. Disponible:
http://www.fpolar.org.ve/ matemática. Extraído: enero 11,
2007

Tales de Mileto (s. VI a.C.), matemático y filósofo de la antigua Grecia, pertenece


al selecto grupo de los siete sabios de la antigüedad, después de ciertos ensayos
y reflexiones, realizó el cálculo de la pirámide de Keops, y lo asombroso fue que
lo hizo sin hacer mediciones de manera directa, ni se esforzó por llegar a la cima
de tan imponente monumento. Se basó en un planteamiento o teorema que lleva
su nombre. Según los antecedentes históricos que se tienen en la actualidad, la
idea de Thales para el cálculo que realizó se
esquematiza como sigue:
Sea “A” la altura de la pirámide para calcular; se
clava una vara, cuya longitud se conoce,
verticalmente donde termina la sombra que
proyecta la pirámide. Los valores “S” y “v” son
las longitudes conocidas de las sombras de la

79
pirámide y de la vara. Luego, mediante el teorema de Thales se puede demostrar
la siguiente igualdad:
D⋅h base
A= , donde D = +S.
v 2

Esa igualdad permite calcular “A”, si se conoce, la base de la pirámide


cuadrangular, la sombra “S” de la pirámide, la altura “h” de la vara y la sombra “v”
de la vara.
Cuando se realizaron los cálculos la asombrosa pirámide tenía 227 m de lado y
146,5 m de altura.
En la actualidad, el procedimiento más común para realizar medidas en cuerpos
geométricos y figuras planas, es la aplicación de fórmulas matemáticas. Esas
medidas que llamamos áreas y volúmenes, no se calculan directamente ya que se
deben medir previamente ciertas magnitudes. Por ejemplo:

Figura Área
Triángulo base ⋅ altura
A=
2

Trapecio B+b
A= ⋅ altura
2

Paralelogramo A = base ⋅ altura

Rectángulo A = base ⋅ altura

Rombo producto _ de _ diagonales


A=
2

Cuadrado A = (lado) 2

Círculo A = π ⋅ r2

Cuerpo Volumen
Prisma recto V = área _ de _ base ⋅ altura

80
Cubo V = (lado) 3

Pirámide área _ de _ base ⋅ altura


V =
3

Cilindro V = π ⋅ r 2 ⋅ altura

Cono π ⋅ r 2 ⋅ altura
V =
3
Esfera 4
V = π ⋅ r3
3

81
LECTURA N° 23: LA TRIGONOMETRÍA
Tomado con fines instruccionales de:
Santamaría, J (2007). La trigonometría. [Artículo no
publicado]. (pp. 1- 3). Tinaquillo, Estado Cojedes.

Es la rama de la geometría, que estudia las relaciones numéricas entre los lados
y los ángulos de los triángulos
. • El origen 0 es el vértice de ángulo y las
semirrectas 0A y 0B son los lados del
ángulo.
• 0A es el lado inicial y 0B es el lado
terminal.
• El ángulo A0B= & se genera mediante la
rotación del lado 0A hasta el lado 0B
• Los ángulos pueden denominarse con
letras del alfabeto griego:
α , β , δ , γ , λ ,σ , φ.
• También puede denominarse ⊄ A0 B ,
que se lee como ángulo A0B.

Un ángulo es positivo si
0A se rota en sentido contrario
al giro de las agujas del reloj
hasta 0B.
Un ángulo, es la posición del plano limitada Un ángulo es negativo
por dos semirrectas que poseen un origen si 0A se rota en el mismo
común. sentido del giro de las agujas del
reloj hasta 0B.

• Un radián es el ángulo central de una • Para convertir de grado a radianes


circunferencia al que le corresponde un multiplicamos el valor del ángulo en
arco de longitud igual al radio. grado por π /180º.
Si 360º=2 π radianes
180º = π radianes de donde • Para convertir de radianes a grado se
1 radián = 180º/ π = 57,30º multiplica el valor del ángulo en
radianes por 180º/ π .

82
Las razones trigonométricas
Consideremos el triángulo rectángulo de referencia

AB = c: Hipotenusa
c BC = a: Cateto opuesto al ángulo &
AC = d: Cateto adyacente al ángulo &
a

&
C d A
Tomando en consideración el triángulo ABC y el ángulo & pueden definirse las razones
trigonométricas en el triángulo rectángulo así:
Se llama seno de & a la razón entre el cateto opuesto BC y la hipotenusa AB:

BC
Sen(&) =
AB

Se llama coseno de & la razón entre el cateto adyacente AC y la hipotenusa AB:


AC
Cos (&) =
AB

Se llama tangente de & a la razón entre el cateto opuesto BC y el cateto adyacente AC:

BC
Tag (&) =
AC

Razones trigonométricas recíprocas


Se llama cotangente de & a la razón entre el cateto adyacente y el cateto opuesto:
AC
Ctg (&) =
CB

Se llama secante de & a la razón entre la hipotenusa AB y el cateto adyacente AC:


AB
Sec(&) =
AC

Se llama cosecante de & a la razón entre la hipotenusa AB y el cateto opuesto BC:

83
AB
Csc (&) =
BC

Identidad fundamental de la trigonometría


Consideremos el triángulo rectángulo mostrado en la figura. Apliquemos el
teorema de Pitágoras a dicho triángulo.
(Hipotenusa)2 = (Cateto)2 + (Cateto)2
De acuerdo al triángulo rectángulo ABC se tiene que:
( AC ) 2 = ( AB ) 2 + ( BC ) 2 ,
luego; dividimos toda la igualdad por (AC)2 y nos
queda:
( AC ) 2 =
( AB ) 2 + ( BC ) 2
,
( AC ) 2 ( AC ) 2 ( AC ) 2
por propiedad de la potenciación, se puede representar así:
2 2 2
 AC   AB   BC 
  =  + 
 AC   AC   AC 
luego; según la definición de las razones trigonométricas se tiene que: Si AC es la
hipotenusa, AB es el cateto opuesto del ángulo α y BC es el cateto adyacente del
ángulo α , entonces:
AB BC
= Senα , = Cosα .
AC AC

AC
Y por propiedad de inverso en la multiplicación = 1,
AC

Por lo tanto, si se sustituye estas igualdades en la anterior, nos queda:


1 = ( Senα ) + ( Cosα ) .
2 2

De esta manera la expresión:


( Senα ) 2 + ( Cosα ) 2 = 1
representa la identidad fundamental de la trigonometría, en función al triángulo
rectángulo y a uno de sus ángulos agudos.
Ejercicios propuestos
23.1- Encierra en un círculo la opción “V” si consideras el enunciado como verdadero o la
letra “F” si lo consideras falso:

84
1. La trigonometría, estudia la simetría de las figuras planas..................... V - F.
2. La identidad fundamental de la trigonometría es llamada teorema
de Euclides
.................................................................................................. ............
V - F.
3. Las razones trigonométricas parten de un triángulo rectángulo.............. V - F.
4. La razón entre el cateto opuesto y el cateto adyacente se llama
cotangente de & de
.................................................................................................. ............
V - F.
5. Para hallar la identidad fundamental hay que aplicar el teorema de
Pitágoras
.................................................................................................. ............
V - F.
6. El seno al cuadrado de un ángulo más el coseno al cuadrado del
mismo ángulo es igual a la unidad
.................................................................................................. ............
V - F.
7. La secante de & es una razón trigonométrica recíproca del coseno....... V - F.
8. El cateto adyacente más el cateto opuesto es igual a la hipotenusa...... V - F.

RESUMEN DE LAS IDENTIDADES TRIGONOMÉTRICAS FUNDAMENTALES


 Las identidades pitagóricas:
1. sen 2 x + cos 2 x = 1 2. tag 2 x + 1 = sec 2 x 3. 1 + cot ag 2 x + 1 = cos 2
 Las identidades del cociente:
senx cos x
4. tagx = 5. cot agx =
cos x senx

 Las identidades recíprocas:


1 1 1
Figura6. 12cos x = 7. senx = 8. cot agx =
senx cos x tgx

Ejercicios:
23.2.Sabiendo que sen α = ¾ calcular el resto de las identidades trigonométricas

3 2
23.3.Dado que la tag & = calcular cos& y sen&
2 3

85
30
23.4.Sabiendo que sec & = calcular sen& y cot&
2 3

m2 − n2
23.5.Sabiendo que cos α = encontrar cot α
m2 + n2
1 tag 2∞ − cos 2 ∞
23.6.Si sen∞ = hallar el valor de la exp resión
2 sec 2 ∞ + cot 2 ∞
sen 2∞ + cot 2 ∞
23.7. sen∞ = 3 hallar el valor de la exp resión + cos 2 ∞
sec ∞ + cos ∞
2 2

1+a
23.8.Dado el triángulo de la derecha, calcular las α

razones trigonométricas del ángulo α 1-a

LECTURA Nº 24: LA TRIGONOMETRÍA ¿PARA QUÉ


SIRVE?
Tomado con fines instruccionales de:
Feria, D. (s.f.) Trigonometría ¿Para qué sirve? [Artículo
en línea]. Disponible:
http://www.es.geocities.com/dferiagomez. Extraido:
diciembre 6, 2007

El problema básico de la trigonometría es algo parecido a esto: Estás cerca de un ancho


río y necesitas conocer la distancia hasta la otra orilla, digamos hasta el árbol marcado en
el dibujo por la letra C (para simplificar, ignoremos la 3ª
dimensión). ¿Cómo hacerlo sin cruzar el río?

La forma habitual es como sigue. Clave dos postes en el suelo


en los puntos A y B, y mida con una cinta la distancia “c”
entre ellos (base del triángulo).
Luego extraiga el poste del punto A y sustitúyalo por un
telescopio de topógrafo "teodolito", contando con una placa
dividida en 360 grados, marque la dirección (azimut) a la que
Figura 10 apunta el telescopio. Dirigiendo el telescopio primero hacia el
árbol y luego hacia el poste B, mide el ángulo A del triángulo
ABC, igual a la diferencia entre los números que ha leído de la placa de azimut. Sustituya
el poste por el teodolito en el punto B y mida de la misma forma el ángulo B. La longitud
“c” de la base y los dos ángulos A y B es todo lo que necesita para conocer el triángulo
ABC, suficiente, por ejemplo, para construir un triángulo de la misma forma y mismo
tamaño, en un sitio más conveniente.

86
La trigonometría (de trigón = triángulo) en un principio, fue el arte de calcular la
información perdida mediante simple cálculo. Dada la suficiente información para definir
un triángulo, la trigonometría te permite calcular el resto de las dimensiones y de ángulos.
¿Por qué triángulos?
Porque son los bloques básicos de construcción para cualquier figura rectilínea que se
pueda construir. El cuadrado, el pentágono u otro polígono puede dividirse en triángulos
por medio de líneas rectas radiando desde un ángulo hacia los otros.
Para medir un terreno, los topógrafos lo dividen en triángulos y
marcan cada ángulo con un "punto de referencia", que hoy en día es a
menudo, una placa de latón redonda fijada en el suelo con un agujero
en el centro, sobre el que ponen sus varillas y teodolitos (George
Washington hizo este trabajo cuando era un adolescente). Después
de medir la base, como la AB en el ejemplo del río, el topógrafo
medirá (de la forma descrita aquí) los ángulos que se forman con el Un antiguo telescopio
punto C y usará la trigonometría para calcular las distancias AC y BC. De topógrafo
(teodolito).
Estas pueden servir como base de 2 nuevos triángulos, que a su vez
suministrarán bases para dos más..., y de esta forma construirá más y más triángulos
hasta que se cubra el terreno completo, con una red que tiene distancias conocidas.
Posteriormente, se puede añadir una red secundaria, subdividiendo los triángulos
grandes y marcando sus puntos con estacas de hierro, que proporcionarán distancias
conocidas adicionales en las que se pueden basar los mapas o los planos.
Un gran proyecto de reconocimiento del siglo XIX fue la "Gran Planimetría
Trigonométrica" de la India británica. Se construyeron para el proyecto los mayores
teodolitos, monstruos con escalas circulares de 36" de ancho, cuyas lecturas se hacían
de manera precisa con 5 microscopios. Cada uno con su caja pesaba media tonelada y
se necesitaban 12 hombres para trasladarlo. Usándolos, el proyecto cubrió el país con
múltiples cadenas de triángulos en las direcciones norte-sur y este-oeste (las áreas entre
las cadenas se dejaron para más tarde) y se necesitaron décadas para completarla.
En 1843 Andrew Scott Waugh, se encargó del proyecto como Inspector General y puso
especial atención a las montañas del Himalaya del norte de la India. Debido a las nubes y
a la niebla, esas montañas se ven raramente desde las tierras bajas, y hasta 1847 no se
consiguieron varias mediciones. Después de haberse hecho, los resultados necesitaron
ser analizados laboriosamente por "computadores" en las oficinas de inspección; no eran
máquinas sino personas que efectuaban los cálculos trigonométricos.
La historia dice que en 1852, el jefe de los "computadores" fue hacia el director y le dijo:
"Señor, hemos descubierto la mayor montaña del mundo". Desde una distancia de más
de 100 millas (160 km), observaron la montaña desde seis estaciones diferentes, y "no
dio lugar a que el observador sospechara que estaba viendo a través de su telescopio el
punto más alto de la Tierra". Al principio se la designó como "Pico XV" por la inspección,
pero en 1856 Waugh la denominó Everest, en memoria de Sir George Everest su
predecesor, en la oficina de jefe de inspectores. El Everest fue el primero en registrarse y
87
en usar los teodolitos gigantes; ahora están expuestos en el "Museum of the Survey of
India" en Dehra Dum.
Hoy en día se puede localizar de forma muy precisa la posición de un punto sobre la
Tierra, usando el sistema de posicionamiento global (GPS) de 24 satélites en órbita
exacta, que están difundiendo constantemente su posición. Un pequeño instrumento
electrónico de mano recibe sus señales y devuelve nuestra posición con un error de 10-
20 metros (aún es más preciso para usos militares, los patrocinadores del sistema). Se
usa una gran cantidad de trigonometría, pero lo hace todo la computadora que está
dentro de su aparato, lo único que usted necesita es pulsar los botones apropiados.

LECTURA Nº 25: TEOREMA DE PITAGORAS


Tomado con fines instruccionales de:
Fundación Polar. Teorema de Pitágoras. [Artículo en línea].
Disponible: http://www.fpolar.org.ve/matemática. Extraído:
enero 4, 2007

En un triángulo rectángulo, el área del cuadrado


construido sobre la hipotenusa es igual, a la suma de
las áreas de los cuadrados construidos sobre los
catetos. Observa cómo los cuadrados construidos
sobre los catetos cubren el cuadrado construido sobre
la hipotenusa.
El cuadrado superior derecho se descompone,
ubicando primero el punto de corte de las diagonales.
Luego se trazan por ese punto, un segmento paralelo
a la hipotenusa y un segmento perpendicular a ella. En
la figura se presenta una “versión visual” de la
comprobación de este teorema.
Observa un ejemplo de comprobación analítica del teorema:
En el triángulo rectángulo que se muestra en la
figura, los lados miden 10, 8 y 6 unidades
respectivamente. Sobre cada lado se ha
construido un cuadrado; según el teorema de
Pitágoras se puede verificar que el área del
cuadrado mayor, es igual a la suma de las
áreas de los cuadrados menores.
Analíticamente hablando, esto es:
( AB) 2 = ( AC ) 2 + ( BC ) 2 , entonces fácilmente
podemos constatar la fórmula sustituyendo los
valores correspondientes.
88
(10) 2 = (8) 2 + (6) 2 , luego 100 = 64 + 36 .

Figura 12

7cm

20cm

35cm

89
UNIDAD 4
PLANO CARTESIANO, RELACIONES Y FUNCIONES
LECTURA Nº 26: EL PLANO CARTESIANO

Tomado con fines instruccionales de:


Santamaría, M. (2006). El plano cartesiano.
Artículo no publicado. (pp.1-2).Tinaquillo,
Estado Cojedes.

La utilidad del plano cartesiano, puede ilustrarse de la siguiente manera: Dos personas
acuerdan encontrarse a las 4:00pm, en cierta esquina de una ciudad cuyo sistema vial
está constituido por calles paralelas y avenidas perpendiculares a las calles, como en el
dibujo.

Calle 5

Cale 4

Cale 3

Cale 2

Avenida Avenida Avenida Avenida Avenida


1 2 3 4 5

Entonces, la ubicación del sitio de encuentro es en la avenida 4 con calle 3.


Si las calles y avenidas no estuvieran numeradas, sino que se identificaran por nombres,
aunque las personas del encuentro no recordaran, sería posible identificar con precisión
el punto de encuentro, si se toma como punto de referencia la catedral, un hospital y una
escuela, por ejemplo:
(3,2)

(0,0)

La ubicación sería: dos cuadras arriba de la iglesia y tres cuadras a la derecha.

90
En este último caso, se usó un sistema para identificar el punto de encuentro, que es
equivalente al sistema de coordenadas cartesianas. Asignando el punto (0,0) a la esquina
de la iglesia; en ese caso, el punto de encuentro tendría coordenadas (3,2), lo que sería
equivalente a decir; tres cuadras a la derecha y dos cuadras hacia arriba.
Ahora veamos cómo se representan estos puntos de referencia matemáticamente en un
Plano Cartesiano.

El plano cartesiano llamado también SistemaY Forma horizontal (eje


de Coordenadas Rectangulares, está formado de las X) se le llama
por dos rectas perpendiculares de origen Eje de Abscisas.
común y dividen el plano en cuatro
cuadrantes.

II CUADRANTE I CUADRANTE
(0,0) origen X
El punto donde se cortan las rectas se llama
Punto de Origen (0,0) y las rectas que se Forma vertical (eje
cortan, las llamamos Ejes de Coordenadas. de las Y) se le
llama Eje de las
Ordenadas.

III CUADRANTE IV CUADRANTE

Las rectas, al cortarse dividen al punto en cuatro partes exactamente iguales. Todo punto
(x,y) lo podemos representar en el plano, mediante un par ordenado de números, donde
la primera componente corresponde a la coordenada “X” y la segunda componente a la
coordenada “Y”
Ejemplo:
Se representa gráficamente el punto “A”, que tiene por coordenadas A (2 ,4)
El par ordenado está en el I cuadrante Y
4
A

2 X

91
LECTURA Nº 27: COORDENADAS Y TECNOLOGÍA

Tomado con fines instruccionales de:


Fundación Polar. Coordenadas y tecnología
[Artículo en línea]. Disponible:
http://www.fpolar.org.ve/matemática/. [Consulta:

Actualmente, la posición de un punto sobre la tierra o un objeto en vuelo en la atmósfera


terrestre, se puede localizar de forma muy precisa mediante el Sistema de
Posicionamiento Global: GPS, por sus siglas en inglés (Global Positioning System),
creado por el Departamento de Defensa de los Estados Unidos en 1978. A partir de 1996
se permitió su uso comercial y civil. Este es un sofisticado sistema de localización de
posiciones, basado en la recepción y procesamiento de las informaciones emitidas,
permanentemente, por una red de 24 satélites que giran en 6 órbitas circulares de 4
satélites cada una, a una altitud aproximada de 20.200km por encima de la superficie
terrestre. Un pequeño instrumento electrónico de mano, denominado GPS, recibe y
procesa la combinación de señales de al menos tres satélites, y muestra la posición en la
que se encuentra respecto a un sistema de coordenadas geográficas que incluyen la
latitud, la longitud y la altura sobre el nivel del mar.
¿Cómo se obtiene la posición de un objeto con el GPS?
Supongamos que uno de los satélites está a una distancia de 15.000km.
Geométricamente, esto indica que el objeto en cuestión, debe estar situado en algún
lugar de la superficie de una esfera, cuyo centro es ese satélite y cuyo radio es de
15.000km. Imaginemos un segundo satélite a 16.000km del objeto que se desea localizar.
Ahora el objeto considerado está, en algún lugar de la superficie de la esfera con centro
en el nuevo satélite y radio 16.000km. En consecuencia, el objeto se encuentra en la
circunferencia intersección de las dos esferas (figura oscura en el gráfico). Pensemos en
un tercer satélite, cuya distancia al objeto en consideración es de 14.000km. La nueva
esfera con centro en ese satélite y radio 14.000km, intersecta a las otras dos esferas en
dos puntos A y B que pertenecen a la figura negra y que señalan la posible ubicación del
GPS. Para saber cuál de los dos puntos señala nuestra verdadera posición, deberíamos
recibir la señal de un cuarto satélite. Pero en la práctica uno de los puntos indica una
posición posiblemente fuera de la Tierra o bien que no cumple con las condiciones
requeridas y por tanto se descarta sin tener que analizar otra nueva señal. El procesador
del GPS realiza instantáneamente los cálculos trigonométricos necesarios para medir la
distancia desde cada satélite y calcular su posición geográfica.

92
LECTURA Nº 28: FUNCIONES QUE TIENEN HISTORIA
Tomado con fines instruccionales de:
Suárez, E. y Cepeda, D. (2003).
Matemáticas de Educación Básica.
Editorial Santillana, S.A. (p.140). Caracas,
Venezuela.

En una oportunidad el rey Hierón encargó a un orfebre la elaboración de una corona de


oro y plata. Lista la corona, el rey le pidió a Arquímedes que comprobara si las cantidades
de oro y plata eran iguales a las que él le entregó al orfebre.
Así llegamos al célebre baño de Arquímedes: al meterse Arquímedes en una bañera llena
de agua, el nivel del agua naturalmente subía, pues cuanto mayor era la parte sumergida
en el líquido, tanto más alto era el nivel de éste. Esto le dio a Arquímedes una gran idea,
llenó completamente un recipiente de agua, sumergió la corona, y recogió el agua que
rebasó el recipiente, ya que él sabía que el exceso de agua debía tener el mismo
volumen que la corona. Lo que quedaba por hacer era sencillo: conseguir una cantidad
similar de oro y comparar su volumen con el de la corona. Así, Arquímedes demostró que
la corona contenía menos oro del acordado.
Actualmente, se pueden comprobar esas relaciones mediante ecuaciones que expresan
funciones: oro: P=19*V; plata: P=(10,5)*V; mercurio: P = (13,6)*V; donde P es el peso y V
es el volumen.

LECTURA Nº 29: LA FUNCIÓN LINEAL

Tomado con fines instruccionales de:


Porras. O. (2004). Tercera Etapa: una
propuesta. Escuela Venezolana para la
Enseñanza de la Matemática. (p.63). Mérida,
Venezuela.
¿Sabías que la notación que se usa actualmente para expresar las ideas relacionadas al
concepto matemático de “función” fue introducida por Leonhard Euler, uno de los más
brillantes genios de la historia de la ciencia?
El concepto de función, ha sido utilizado desde entonces prácticamente en todas las
ramas de la matemática, dicho concepto matemático permite organizar información que
se obtiene a través de datos numéricos tomados de algún fenómeno, y estudiar la
manera en que esos datos se relacionan entre sí. Por ejemplo, se tienen los siguientes
datos acerca de los kilómetros recorridos por un ciclista en entrenamiento, en intervalos
de tiempo de 15 minutos:
Minutos 0 15 30 45 60
kilómetros 0 6 12 18 24

93
Un observador cuidadoso notará que, en cada intervalo de 15 minutos, el número de
kilómetros avanzados es siempre el mismo, 6km. Si se representan estos datos en el
plano cartesiano, ubicando el tiempo en minutos en el eje de las abscisas (x) y la
distancia recorrida en el eje de las ordenadas (y), se obtiene algo así:

(60,24)
24

(45,18)
18

(30,12)
12

(15,6)
6

0 15 30 45 60

Esto indica que el ciclista va a una velocidad constante, y que una línea recta representa
su recorrido en kilómetros a través del tiempo.
El tiempo y la distancia se denominan variables. El tiempo es, en este caso la variable
independiente y la distancia recorrida es la variable dependiente, porque depende del
tiempo: para cada instante dado, hay una distancia recorrida.
Una función, es una manera de asociar cada elemento de un conjunto de variables con
un elemento de otro conjunto de variables (como en este caso) y se escribe f (x) para
representar el número que se le asocia a la variable independiente x .
f (0) = 0 ; f (15) = 6 ; f (30) = 12 ; f (45) = 18 ; f (60) = 24

2
Y en general: f ( x) = .x
5

Es decir, cada vez que la x (variable independiente) aumenta en 15 minutos, la variable


dependiente (F(x) o y) aumenta en 6 kilómetros.

94
LECTURA Nº 30: DISTANCIA ENTRE DOS PUNTOS EN
EL PLANO
Tomado con fines instruccionales de:
Santamaría, M. (2007). Distancia entre dos
puntos en el plano. Artículo no publicado.
(pp.1-2). Tinaquillo, Estado Cojedes.

RECUERDA La distancia entre dos puntos en el plano, se puede calcular


Teorema de
Pitágoras en todo aplicando el Teorema de Pitágoras en función de las
triangulo rectángulo, coordenadas de esos puntos.
la longitud de la
hipotenusa al En la figura se ha formado el triángulo ABC rectángulo en C ,
cuadrado es igual a donde la medida de la hipotenusa AB corresponde a la distancia
la suma de los entre los puntos A( x1 , y1 ) y B( x2 , y2 ), que se designa como
cuadrados de las
longitudes de los
d ( AB ) .
catetos.
( AB ) 2 = ( AC ) 2 + ( BC ) 2
B
d ( AB ) 2 = ( x2 − x1 ) 2 + ( y2 − y1 ) 2
a c
Por lo tanto d ( AB) = ( x 2 − x1 ) 2 + ( y 2 − y1 ) 2
C A De forma tal que: dados dos puntos con sus respectivas
b coordenadas, se puede encontrar la distancia entre ellos sólo con
aplicar la fórmula correspondiente.
c2 = a2 + b2

ACTIVIDADES: Hallar la distancia entre los puntos:


30.1- A(-2,3) y B(-2,1) 30.2- C(4,4) y D(7,-9) 30.3-E(8,-1) y F(-6,-5)
30.4- G(-5,3) y H(6,6) 30.5- I(4,-3) y J(2,9) 30.6-K(1,1) y U(9,4)
30.7-Representa gráficamente cada par de puntos.

95
Ejemplo: ¿Cuál es la distancia 30.8) En la siguiente gráfica obtener:
entre los puntos A(-9, 7), y a) Todos los pares ordenados.
B(3, 2)? Aplicando la fórmula b) Todas las combinaciones posibles entre dos
respectiva, se obtiene: puntos.
c) Hallar la distancia entre dos puntos de las
d ( AB) = [ 3 − (−9)] 2 + (2 − 7) 2 combinaciones obtenidas.
= (3 + 9) 2 + (2 − 7) 2
= (12) 2 + (−5) 2
= 144 + 25 3
= 169
= 13 2

Y 1
7
0
-2 -1 1 2 3
2 -1

-9 3
X -2

30.9-Si un triángulo tiene como vértices los puntos: A(-2,-3); B(-1, 1) y C(0,4). Calcula
el perímetro del mismo.
30.10-Del ejercicio anterior, calcula los puntos medios de cada lado y entre estos
puntos traza un nuevo triángulo. Calcula el perímetro del nuevo triángulo.
30.11-¿Qué relación existe entre los perímetros calculados en los ejercicios 30.8 y
30.9?
30.12-Si las coordenadas del punto A son (3,4) y la distancia de A hasta B es 5.
Indica dos de las posibles coordenadas de B.

96
LECTURA Nº 31: CLASIFICACIÓN DE LAS
FUNCIONES
Tomado con fines instruccionales de:
Santamaría, M. (2007). Clasificación de las
funciones. Artículo no publicado. (pp.1-2).
Tinaquillo, Estado Cojedes.

Se llama Función Lineal f: R- R definida por la ecuación de primer grado f ( x ) = ax + b,


donde a y b son constantes.
La gráfica de la Función Lineal es una línea recta.
La Ecuación de la recta puede explicarse en forma implícita o en forma explícita

Donde Donde

Representación gráfica de la función lineal


Para representar gráficamente una Función Lineal se requiere conocer, por lo menos, dos
puntos de ella.
Estos puntos se llevan al plano de coordenadas y se unen a través de una línea recta. La
recta trazada será la gráfica de la función lineal dada.
Ejemplo:
Representar gráficamente la función Y = 2 x − 1 , le damos valores arbitrarios a la variable
“X” en la tabla.
5
4
3
2
1

-2 -1 1 2 3 4 5 6 7

Ejercicios:
Completa las siguientes tablas según las funciones reales dadas. Luego, grafícalas y dale
el nombre a cada una.
x
31.1) f ( x) = +1 31.2) g ( x) = 6 x − 3
3

97
Representa gráficamente la Función

−2 1 2 7 5
31.3) X + Y− =0 31.4) Y ) = X−
3 2 3 3 3

Pendiente de la recta
La tangente trigonométrica del
ángulo que forma dicha recta con
el eje horizontal del sistema. Se
La pendiente (m) denota con la letra (m).
de la recta y la
ordenada del La tangente de un ángulo es igual
origen (b) quedan al cateto opuesto sobre cateto
perfectamente adyacente:
definidos en la
fórmula de la Tag α = CO m = CO
función lineal CA CA
correspondiente:
y 2 − y1
m=
x 2 − x1

Ejemplo:
Determina la pendiente de la recta que pasa por los puntos A(2,4) y B (4,6)

x y
2 4 y 2 − y1 6 −4 2
m= m= = =1 ⇒m =1
4 6 x 2 − x1 4 −2 2

Determina la pendiente de la recta y = 3 x + 2 que pasa por los puntos


A(2,8) y B (−3,−7) NOTA: No era necesario aplicar la fórmula ya que
en la función lineal f ( x) = mx + b el coeficiente de
la variable x es la pendiente.
x y
− 7 − 8 − 15
2 8 m= = =3⇒ m =3
−3− 2 −5
-3 -7

98
Ejercicios:
Despeja la “Y”, e indica la pendiente de cada recta.
31.5) 3 x + y = 5 31.6) 2 x − 3 y = 6 31.7) − 2 y = x
Calcula la pendiente de las siguientes rectas dadas por dos puntos.
31.7) AB si A(3,4) y B(5,7) 31.8) CD si C ( −1,6) y D ( −3,5)
31.9) AB si A( −2,−2) y B (2,2)
Despeja la “Y”, e indica la pendiente de cada recta.
x+ y
31.10) y = 31.11) 4 x + 2 y − 7 = 0 31.12) 4 − y = 0
2
Calcula la pendiente de las siguientes rectas dadas por dos puntos.
31.13) MN si M (0,1) ∧ N (4,−2) 31.14) TU si T (2,1) ∧U ( 2,−3)
Resuelve los siguientes problemas:
31.15) Sabiendo que una persona acorta su vida en 8 minutos cada vez que se fuma un
cigarrillo. Expresa en forma de función, el cálculo de la cantidad de horas que una
persona acorta su vida en un mes si se fuma x cantidad de cigarrillos al día.
31.16) Elabora la gráfica para la función del ejercicio 31.15 y calcula la pendiente.
31.17) Dibuja la gráfica del paralelogramo cuyos vértices están representados por los
puntos: A(-3,-1); B(0,-4); C(6,4) y D(3,7). Utilizando el criterio del cálculo de la
pendiente de los lados, determina que figura se forma al unir los puntos medios de
los cuatro lados.
Función cuadrática
Ya has estudiado la función lineal, la cual se puede definir como una relación entre dos
conjuntos numéricos, un conjunto dominio (valores que puede tomar la x) y un conjunto
rango (valores de la y). La relación entre estos conjuntos, está dada de forma que cada
elemento del dominio le corresponde una imagen o elemento siempre distinto en el
rango. Es decir, la función lineal pertenece al grupo de las funciones biyectivas. Se
caracteriza por tener como ecuación la forma y = mx + b y su gráfica es una recta en
el plano cartesiano.
Ahora bien, la función cuadrática también
es una relación entre dos conjuntos
numéricos, pero aquí la correspondencia
entre los elementos, es un poco diferente a la
que se presenta en la función lineal; debido a
que pares de elementos distintos en el
conjunto dominio se relacionan con una
misma imagen o elemento en el conjunto
rango. La función cuadrática podría ser un

99
típico caso de las funciones sobreyectivas.
Se caracteriza por tener como ecuación la
forma y = Ax 2 + Bx + C ; siendo los
coeficientes A, B y C números reales
cualesquiera y A ≠ 0 , y además su gráfica
representa una parábola o curva abierta en
forma de campana, simétrica respecto a un
eje.

Veamos la comparación entre la función lineal y la cuadrática.


Función Lineal Función Cuadrática
Ecuación y = f ( x) = mx + b y = f ( x) = Ax 2 + Bx + C
Variable La variable “X” tiene exponente 1 La variable “X” tiene exponente 2
Gráfica Es una línea recta Es una parábola
Tipo de Es Inyectiva y sobreyectiva Puede ser sobreyectiva
función

Funciones reales

FUNCIÓN: Cuando una cantidad variable depende de otra


se dice que está en función de esta última.
En una definición moderna de función, Cauchy, explica que
“Y” es función de “X” cuando el valor de la variable X le
corresponden uno o varios valores determinados de la
variable “Y”.
La notación para expresar que Y es función de X es:

Una Función
Real, es una
función cuyo Hemos estudiado las
dominio y rango Funciones Lineales
están constituidos Funciones Cuadráticas
por números Funciones Reales
reales R o
subconjuntos de
éste.
100
101
Ejercicios:
Determina cuáles de las siguientes funciones son cuadráticas:
31.18- y = 2 x + 5 31.19- y = 3 x − 5 x + 6
2 2

31.20- y = 2 x − 1 31.21- y = ( x + 2).( x − 2 )

 x 1
2
31.23- y = x + 5x − 1
31.22- y =  + 
 2 3
31.24-¿Cuáles son los coeficientes de las funciones cuadráticas de los ejercicios
36.18 al 36.23
31.25-Realiza la gráfica de las funciones del ejercicio 34.24
Desde la azotea de un edificio de 30m. de altura, es lanzado hacia arriba, un
cohetón (fuegos artificiales). El recorrido del artefacto se expresa mediante la
función: f (t ) = −2t + 32t (siendo t el tiempo).
2

31.26-Determina la altura del cohetón al transcurrir 1seg, 2seg, 3seg, 4seg y 5 seg.
31.27-Elabora una gráfica de la función.
31.28-¿Cuál es la mayor altura que alcanza el artefacto y a cuántos segundos?
31.29-¿A cuántos segundos pasa en caída frente al mismo lugar de donde fue
lanzado?
31.30-¿Cuántos segundos demora en caer al nivel de la carretera?

Función exponencial
Una Función Exponencial, es una función de la forma y = f ( x) = a , donde a es un
x

número real positivo distinto de 1. De acuerdo a esto, podemos escribir en símbolos lo


siguiente:

Una función de la forma y = f ( x) = a x , donde a > 0 y a ≠ 1 , es una


función exponencial, siendo “a” la base de la función exponencial.

Ejemplo:
x x
1 1
y=2 x
y =5 x
y=  y = 
2 5

NOTA: Debemos hacer notar que si a = 1 , entonces a x se transforma en 1x = 1 y se


tendría una función constante. Es ésta, la razón por la cual se impone en la definición
que a ≠ 1 .
102
Ejemplo: Gráfica de f ( x) = 2
x

Tabla de Datos

De la gráfica podemos observar varios


aspectos:
Cuando X aumenta ( x → +∞ ) los valores
de Y aumentan con rapidez, mientras que
cuando los valores de X disminuyen
( x → −∞ ) los valores de Y se acercan
cada vez más a “0”. En este caso se dice
que el eje X es una Asíntota Horizontal.
La función es creciente.
Por otro lado, no existen intersecciones
con el eje X porque b x ≠ 0 para
cualquier valor de X.
La intersección con el eje Y es el punto
(0,1) ya que b0=1.

Propiedades de las funciones exponenciales


• El dominio de la función exponencial es el conjunto de todos los números reales.
• El rango de la función exponencial es el conjunto de todos los números reales
positivos (R+).
• La gráfica de Y = a x muestra un crecimiento exponencial si a > 1 Función
Creciente.
• La gráfica de Y = a x muestra un decrecimiento exponencial si 0 < a < 1 Función
Decreciente.
• La intersección con el eje Y es 1, no existiendo intersección con el eje X.
• El eje X es una Asíntota Horizontal.
• A mayor valor de a , mayor será la rapidez con que crece la función.
• La gráfica de cualquier función exponencial pasa por el punto (0,1) porque a0 = 1 .
• Por ser a1 = a , la gráfica pasará siempre por el punto (1,a).
• Es inyectiva y sobreyectiva, razón por la cual es biyectiva.

103
Ejercicios:
Calcula los valores que toman las siguientes funciones para X = -2,-1, 0, 1, 2.
31.31- f ( x) = 5 x 31.32- f ( x) = 5− x
x −x
1 1
31.33- f ( x) =   31.34- f ( x) =  
5 5
31.35-En la definición de función exponencial la base a fue restringida
(a > 0) ∧ (a ≠ 1) .
31.36-Si aceptamos la condición a = 1 ¿Qué le sucede a la función Y = a x ?
31.37-Si el precio de un producto crece de acuerdo a las funciones
Y = 3 x y Y = 3x ¿Cuál de las funciones nos conviene si somos
compradores? Razona tu respuesta

La Función Logarítmica
La expresión X = 3 y significa que “Y” es el
exponente al que es necesario elevar la base
Hemos estudiado que la función
3 para obtener “X”.
exponencial Y = a x (a > 0 ya ≠ 1) es
Entonces se dice que:
biyectiva y como consecuencia tiene
Para a > 0 y a ≠ 1 el logaritmo de base
una función inversa. Como por
“a” de un número X>0 es el exponente al que
ejemplo Y = 3 x su inversa X = 3 y
imposible despejar a “Y”. hay que elevar la base para obtener dicho
número.

log a X = Y Es equivalente a aX = a y
Y = log a X Se lee: “Y es igual a logaritmo de X en la base a” o
“Y es igual al logaritmo base a de X”

La siguiente tabla ilustra la equivalencia Consecuencias inmediatas de la


definición de logaritmo:
de las formas exponenciales y logarítmicas.
Forma Exponencial Forma Logarítmica 1. X = ay si y sólo si Y = logax

32 = 9 log 3 9 = 2 2. alogaX = X

33 = 9 log 2 8 = 3 3. logaay = Y

30 = 1 log 3 1 = 0 4. loganam = m/n

104
Ejercicios:
Indica en forma de función logarítmica las Indica en forma de función exponencial las
siguientes expresiones: siguientes funciones logarítmicas
31.38-hk = p 31.45-loghp = k
31.39-72 = 49 31.46-logph = k
31.40-(1/3)4 = 1/81 31.47-log249 = 7
31.41-27-1/3 = 1/3 31.48-log749 = 2
31.42- ( 2) x
= 1024 31.49- log 2 1024 = x

31.43- ( 2 ) 31.50- log x 2 = 1024


x
= 16
−3 1 1
31.44- 4 = 31.51- log −3 = −3
64 64

Funciones Trigonométricas
Las funciones trigonométricas se obtienen a partir de las razones trigonométricas, de la
forma siguiente:
• El ángulo se expresa en radianes. Por tanto, los 360º de una circunferencia pasan a
ser 2 π radianes
• Se considera que cualquier número real puede ser la medida de un ángulo. Sus
razones trigonométricas se relacionan con las razones de los ángulos comprendidos
en el intervalo [ 0,2π ) del siguiente modo:
Si x-x1 = k2 π , con k número entero, entonces
sen(x) = sen(x)1 cos(x) = cos(x)1 tg(x) = tg(x)1
es decir, si dos números difieren en un número entero de veces 2 π , entonces tienen los
números razones trigonométricas.
De este modo se obtienen las funciones trigonométricas, llamadas también circulares:
Y = sen(x), Y = cos(x) Y =t g(x),
Sus representaciones gráficas son:

105
Gráfica de la función seno
Formamos una tabla de valores
Radianes Sen x
0 0
π 1
2
π 0
3π -1
2
2π -0,87
π -1
-
2

Análisis de la gráfica
• A medida que el ángulo crece de 0 a π /2, los valores del seno crecen de 0 a 1; por lo
tanto la curva es creciente en este intervalo y sus valores son positivos. El máximo
ocurre cuando X= π /2 .
• A medida que el ángulo crece de π /2 a π , los valores del seno varían de 1 a 0. en
este intervalo la curva es decreciente y sus valores son (+).
• A medida que el ángulo crece entre π y 3 π /2 los valores del seno varían de 0 a -1
en este intervalo la curva es decreciente y sus valores se obtienen cuando X=3 π /2.
• A medida que el ángulo crece entre 3 π /2 y 2 π los valores del seno varían -1 y 0; por
lo tanto, la curva es creciente y sus valores son negativos.
• La función sen(x) es contraría para el intervalo 0 a 2 π . Esto nos indica que no tiene
roturas en su gráfica.

Gráfica de la Función Coseno


La función coseno es una función real de variable real, tal que a cada ángulo & medido
en radianes se le hace corresponder un número real denominado como cos&.

106
Formamos una tabla de valores.
Radianes Cos x
0 1
π 0
2
π -1
3π 0
2
2π 1
π 0
-
2

La función tangente:
La función tangente es una función de variable real definida como el cociente
senx
f ( x) =
cos x

siendo cos(x) diferente de cero (0), denotado por f ( x) = tagx , de forma tal que a cada
ángulo expresado en radianes, le haga corresponder el valor de su tangente.
Gráfica de la función tangente
Formamos una tabla de valores.
Radianes Cos x
0 0
π 1
4
π 0
3π -1
4
−π 0

107

You might also like